4280 Hesi Questions

¡Supera tus tareas y exámenes ahora con Quizwiz!

While caring for a formula-fed 2-day-old, which nursing interventions promote safe sleep? (MA) A. Keeping bassinet free of toys and supplies B. Placing the newborn prone to sleep C. Offering a pacifier during sleep D. Encouraging the family to bed-share

A. Keeping bassinet free of toys and supplies C. Offering a pacifier during sleep E. Dressing the newborn in a sleep sack

Which duties are appropriate for a support person during labor? (MA) A. Keeping patient calm B. Keeping track of contractions C. Interpreting fetal monitoring D. Administering pain medication E. Ensuring physical and emotional comfort

A. Keeping patient calm B. Keeping track of contractions E. Ensuring physical and emotional comfort

A woman in labor has been having regular contractions but has remained 5 cm dilated for 5 hours, with a reassuring fetal heart rate. Which intervention may be necessary for this patient? A. Labor augmentation B. Cesarean delivery C. Vacuum-assisted delivery D. Intrauterine resuscitation

A. Labor augmentation

Which effect does secretion of human chorionic gonadotropin have on the endometrial cycle of the female reproductive cycle? A. Lack of menstruation B. Release of multiple ova C. Necrosis of the endometrium D. Decreased levels of follicle-stimulating hormone (FSH)

A. Lack of menstruation

Which characteristics predispose the newborn to heat loss? (MA) A. Large body surface-to-mass ratio B. Thin subcutaneous fat layer C. Flexion of extremities D. Maternal temperature at birth E. Superficial blood vessels

A. Large body surface-to-mass ratio B. Thin subcutaneous fat layer E. Superficial blood vessels

Which criterion is used to classify an infant as small-for-gestational-age (SGA)? A. Less than 10th percentile for weight B. 10th percentile for height C. Less than 37 weeks' gestation D. Less than 25th percentile for weight

A. Less than 10th percentile for weight

Which advantages of a low transverse skin incision would a nurse explain to a patient? (MA) A. Less visibility when sealed B. Less chance of wound dehiscence C. Pubic hair will grow back D. Works well with obese patients E. Allows the surgeon to extend the incision upward F. Quicker to perform

A. Less visibility when sealed B. Less chance of wound dehiscence C. Pubic hair will grow back

A 54-year-old woman with a 20-year history of smoking and a family history of myocardial infarction reports being "prediabetic" and does not exercise regularly. Which screening procedure would the nurse identify as a priority for this woman? A. Lipid profile B. Mammography C. Bone density scan D. Fasting glucose test

A. Lipid profile

Which method of anesthesia in labor is considered safest for the fetus? A. Local infiltration B. Epidural block C. Spinal block D. Combined spinal-epidural (CSE) block

A. Local infiltration

Which situations are considered risk factors for a postpartum mood disorder when they are present before the pregnancy? (MA) A. Low socioeconomic status B. Strong social support C. Infertility treatments D. History of depression E. 27 years old at time of conception

A. Low socioeconomic status C. Infertility treatments D. History of depression

A nurse is on the phone with a woman 3 weeks after her intrauterine device (IUD) insertion. Which symptom should be reported to the woman's health care provider immediately? A. Malodorous vaginal discharge B. Strings that can be felt during intercourse C. Stronger cramps during menstruation D. Strings that are the same length as the day of insertion

A. Malodorous vaginal discharge

Which major concern regarding epilepsy in pregnancy would the nurse recognize? A. Many antiseizure medications are known teratogens. B. There is an increased risk for miscarriage and preeclampsia. C. A woman who has been seizure-free for many years may have a severe exacerbation in pregnancy. D. Epilepsy in pregnancy may lead to Graves disease.

A. Many antiseizure medications are known teratogens.

A laboring patient reports moderate back pain. Her partner rubs her lower back and reminds her to look at the teddy bear they bought for the newborn. Which relaxation techniques are being used? (MA) A. Massage B. Focal point C. Hydrotherapy D. Acupressure E. Mental imagery

A. Massage B. Focal point

Which assessment is most important to monitor after the administration of epidural anesthesia? A. Maternal blood pressure (BP) B. Fetal Bp C. PLatelet count D. Respiratory rate

A. Maternal blood pressure (BP)

A pregnant woman is yawning excessively, is diaphoretic, and is experiencing nausea, vomiting, diaphoresis, and diarrhea. The nurse suspects that the woman may be withdrawing from which substance? A. Opioids B. Alcohol C. Cocaine D. Marijuana

A. Opioids

Which statements describe how the uterus, ovaries, and fallopian tubes function together to assist in conception? (MA) A. Ovaries mature the ovum. B. The uterus stores glycogen. C. Fallopian tubes transport the ovum. D. The uterus supports pelvic structures. E. The ovaries release follicle-stimulating hormone (FSH) and luteinizing hormone (LH).

A. Ovaries mature the ovum. B. The uterus stores glycogen. C. Fallopian tubes transport the ovum.

Which statements regarding prostaglandins (PGs) are accurate? (MA) A. PGs are produced by most organs of the body. B. PGs have an effect on ovulation. C. PGs influence production of estrogen. D. PGs are produced only in the uterus during pregnancy. E. PGs are produced by the graafian follicle.

A. PGs are produced by most organs of the body. B. PGs have an effect on ovulation. C. PGs influence production of estrogen.`1

Which patient cues would be cause for concern when interviewing a patient regarding her intrauterine device that was placed 3 months ago? (MA) A. Patient reports her partner tested positive for chlamydia. B. A patient reports an unexplained fever and chills. C. The patient can feel the strings in her vagina with her fingers. D. The patient reports pelvic pain. E. The patient reports abnormal vaginal discharge.

A. Patient reports her partner tested positive for chlamydia. B. A patient reports an unexplained fever and chills. D. The patient reports pelvic pain. E. The patient reports abnormal vaginal discharge.

A G1/P0 gestational diabetic mother is undergoing induction of labor. She is in her 39th week of gestation, and she has been diagnosed with polyhydramnios. The nurse recognizes which patient cue as a risk factor for umbilical cord prolapse? A. Polyhydramnios B. Being a primigravida C. Gestational diabetes D. Term gestation

A. Polyhydramnios

The nurse assessing for hypoglycemia in a neonate would monitor for which symptoms? (MA) A. Poor feeding B. Flexion of the arms and legs C. Hypothermia D. Tremors E. Bluish hands and feet F. Rooting

A. Poor feeding C. Hypothermia D. Tremors

Which underlying conditions are associated with amenorrhea? (MA) A. Pregnancy B. Endometriosis C. Tubal ligation following childbirth D. Cocaine use E. Stress

A. Pregnancy D. Cocaine use E. Stress

Which would the nurse assess at a routine prenatal visit for a patient that has gestational diabetes? (MA) A. Presence of bacteria, ketones, protein, and glucose in urine B. Fetal movement by palpation and patient report C. Serum calcium levels D. Blood pressure E. Fundal weight

A. Presence of bacteria, ketones, protein, and glucose in urine B. Fetal movement by palpation and patient report D. Blood pressure E. Fundal weight

A 36-week gestation patient presents with membranes grossly ruptured and is not contracting. Which diagnosis does the nurse anticipate? (MA) A. Preterm premature rupture of membranes (PPROM) B. Preterm labor C. Premature rupture of membranes (PROM) D. AROM

A. Preterm premature rupture of membranes (PPROM)

Which conditions are possible causes of dysfunctional labor? (MA) A. Psychological dysfunction and fear B. Absence of a void in 6 hours C. Rapid descent of the fetal head and small parts D. An abnormally shaped maternal pelvis E. Meconium-stained amniotic fluid F. Use of oxytocin to augment labor

A. Psychological dysfunction and fear B. Absence of a void in 6 hours D. An abnormally shaped maternal pelvis

Which nursing action is appropriate for a menopausal woman who tells the nurse that she finds intercourse uncomfortable? A. Recommend the use of water-based lubricants before intercourse. B. Recommend abstinence from intercourse during menopause. C. Recommend the use of a condom during intercourse. D. Recommend an increase in fluid intake.

A. Recommend the use of water-based lubricants before intercourse.

When administering nifedipine for tocolysis, it is important for the nurse to monitor for which serious side effect? A. Reflex tachycardia B. Hyperglycemia C. Change in fundal height D. Bradypnea

A. Reflex tachycardia

After assessment of an actively laboring woman, the nurse finds a fetal heart rate (FHR) of 180 beats/min with contractions occurring less than 2 minutes apart and lasting more than 90 seconds. Which action would the nurse take next? A. Reposition the patient to her side and alert the health care provider. B. Nothing, these are normal findings. C. Prepare for emergent cesarean delivery. D. Reposition the patient only.

A. Reposition the patient to her side and alert the health care provider.

A nurse caring for a patient immediately postpartum after a precipitate labor would monitor the patient for which possible postpartum complication related to her precipitate labor? A. Retained placenta B. Infection C. Low apgar scores D. Postpartum depression

A. Retained placenta

Which intervention is appropriate for a patient with Rh-negative blood who is unsensitized and just received percutaneous umbilical blood? A. Rho(D) immune globulin administration B. Transvaginal ultrasound C. Transabdominal ultrasound D. Anticoagulant administration

A. Rho(D) immune globulin administration

The nurse understands that which hormonal changes are thought to encourage the onset of labor? (MA) A. Rising estrogen counteracts the relaxing effect of progesterone on the uterus. B. Follicle-stimulating hormone is secreted in large quantities by the anterior pituitary. C. Prostaglandins are secreted by the fetal membranes preparing the uterus. D. The fetus secretes cortisol, which may act as an additional uterine stimulant. E. Thyroid-stimulating hormone is enhanced and helps make the uterus contractile. F. Oxytocin is secreted by the mother, which causes uterine contractions.

A. Rising estrogen counteracts the relaxing effect of progesterone on the uterus. C. Prostaglandins are secreted by the fetal membranes preparing the uterus. D. The fetus secretes cortisol, which may act as an additional uterine stimulant. F. Oxytocin is secreted by the mother, which causes uterine contractions.

At which stage in Rubin's tasks of pregnancy does the pregnant woman typically begin prenatal care? A. Seeking safe passage B. Gaining acceptance of others C. Binding-in to unborn child D. Giving of oneself

A. Seeking safe passage

Which are the advantages of external electronic fetal monitoring (EFM)? (MA) A. Suitable for most patients in labor B. Visualization of heart rate pattern C. Allows measurement of contraction intensity D. Uninterrupted by fetal or patient movement E. Allows mobility if connected to mobile unit

A. Suitable for most patients in labor B. Visualization of heart rate pattern E. Allows mobility if connected to mobile unit

Which observations suggest that a woman is the latent phase of the second stage of labor? (MA) A. The fetus is at 0 station. B. There is an irregular and inconsistent urge to bear down with contractions. C. The woman exhibits loud, songlike vocalizations with every contraction. D. The fetus is at +3 station. E. The woman sleeps between contractions. F. The woman is pushing involuntarily.

A. The fetus is at 0 station. B. There is an irregular and inconsistent urge to bear down with contractions. E. The woman sleeps between contractions.

Which statement regarding pain experienced during labor is accurate? A. The predominant pain of the first stage of labor is the visceral pain located in the lower portion of the abdomen. B. Referred pain is the extreme discomfort between contractions. C. The somatic pain of the second stage of labor is more generalized and related to fatigue. D. Pain during the third stage is a somewhat milder version of the second stage.

A. The predominant pain of the first stage of labor is the visceral pain located in the lower portion of the abdomen.

How is effacement measured? (MA) A. Through digital palpation by a trained professional B. Using transvaginal ultrasound C. By estimating the stage of labor D. Though palpation of cervical dilation E. Through Leopold maneuvers

A. Through digital palpation by a trained professional B. Using transvaginal ultrasound

The antepartum nurse would provide the pregnant woman education on which signs and signals to report immediately to her health care provider? (MA) A. Vaginal bleeding B. Rupture of membranes C. Heartburn accompanied by a severe headache D. Fetal movement E. White, acidic discharge

A. Vaginal bleeding B. Rupture of membranes C. Heartburn accompanied by a severe headache

Which clinical finding is considered a positive sign of pregnancy? A. Visualization of the fetus by ultrasound B. Positive serum pregnancy test C. Breast enlargement D. Braxton Hicks contractions

A. Visualization of the fetus by ultrasound

Which nonmedical therapy could be recommended for a woman experiencing infertility? A. Weight loss of 15 lb B. Assisted reproductive technology (ART) C. Infertility support group D. Vegan diet

A. Weight loss of 15 lb

A patient with preeclampsia has an emergency delivery at 32 weeks gestation. Which findings would the nurse anticipate in this premature neonate? (MA) A. Yellow skin B. Weight of 2000 g C. Axillary temperature of 97.7°F D. Heart rate of 120 beats/min E. Poor respiratory effort

A. Yellow skin B. Weight of 2000 g E. Poor respiratory effort

A patient asks the nurse to describe which factors are included in HELLP syndrome. Which factors would the nurse include in response? (MA) A. low platelets B. Hemolysis C. Low leukocytes D. Elevated erythrocytes E. Elevated liver enzymes

A. low platelets B. Hemolysis E. Elevated liver enzymes

Which nursing response is appropriate for the mother of a 12-year-old who is concerned that her daughter has not started her period? A. "Your daughter may be experiencing primary amenorrhea. Please make an appointment with the health care provider as soon as possible." B. "Although the average age of menses is 12 years, some girls may not begin menses until age 16 years." C. "Your daughter may be experiencing dysmenorrhea and should be evaluated by the health care provider as soon as possible." D. "Your daughter should start her period at the same age that you started your period. What age did you begin menstruating?"

B. "Although the average age of menses is 12 years, some girls may not begin menses until age 16 years."

A woman had an emergency cesarean delivery for a prolapsed cord. Both the mother and infant are stable, and the nurse is caring for the mother and infant couplet. Which statement by the mother indicates she is in the taking-in phase? A. "Do you think the baby is breathing OK? His breathing seems very fast, and then it slows down. Is this normal?" B. "I can't wait for my sister to come with food and take pictures of the baby while I rest." C. "Not feeling movement in my stomach seems so different now. I didn't know how much the baby moved when I was pregnant." D. "The baby is so small that I worry that I will hurt him when I change his diaper. He moves so much!"

B. "I can't wait for my sister to come with food and take pictures of the baby while I rest."

A patient with a 2-year-old child with cystic fibrosis declines genetic counseling during her antepartum visit, stating that the birth defect has "already happened," and therefore her future children will not be affected. Which is the appropriate nursing education for this patient? A. "The risk for a child developing cystic fibrosis increases with maternal age." B. "This infant has an equal risk for developing cystic fibrosis as your first child." C. "The risk for your infant developing cystic fibrosis is less than your first child, but it is still present." D. "There is no longer a risk for your infant developing cystic fibrosis, but genetic counseling can help detect other abnormalities."

B. "This infant has an equal risk for developing cystic fibrosis as your first child."

Which nursing recommendation is appropriate for a woman experiencing pain associated with endometriosis? A. "Take oral oxycodone/acetaminophen for pain every 4 hours around-the-clock." B. "You may take regular strength ibuprofen for pain every 8 hours." C. "The use of laxatives will help reduce the pain." D. "You should not take any medications when you have pain associated with endometriosis because they may cause addiction."

B. "You may take regular strength ibuprofen for pain every 8 hours."

Which woman has an increased risk for developing preeclampsia? A. A 25-year-old woman with liver disease B. A 26-year-old woman with a body mass index (BMI) of 32 C. A 34-year-old woman who has breast cancer D. A 30-year-old woman with a family history of diabetes

B. A 26-year-old woman with a body mass index (BMI) of 32

A nurse is caring for a patient in labor who is receiving oxytocin via intravenous infusion to stimulate uterine contractions. Which assessment finding would indicate to the nurse that the infusion needs to be discontinued? A. Three contractions occurring within a 10-minute period B. A fetal heart rate of 90 beats/min C. A resting uterine tone of 15 mm Hg via the intrauterine pressure catheter D. Early decelerations

B. A fetal heart rate of 90 beats/min

What would the nurse expect when caring for a full-term primigravida who presents for a routine office visit and is not in labor? A. A fundal height that measures 30 cm B. A fetus that is engaged in the maternal pelvis C. A cervix that is already 3 cm dilated D. Fetal station of −3

B. A fetus that is engaged in the maternal pelvis

An Rh-negative woman needs an amniocentesis. The nurse understands which statement to be true based upon the information provided? A. The amniocentesis will determine if the mother needs a blood transfusion. B. A shot of Rho(D) immune globulin will be required for this procedure. C. A Coombs test should be performed immediately after the procedure. D. The fetus is likely Rh-positive, so delivery should be expedited.

B. A shot of Rho(D) immune globulin will be required for this procedure.

A nurse on the postpartum unit is caring for several postpartum women. Which woman would the nurse recognize as having the greatest risk for developing a postpartum infection? A woman who experienced a precipitous labor less than 3 hours in duration B. A woman who has type 1 diabetes and delivered a large-for-gestational-age infant via cesarean section C. A woman who had a boggy uterus that was not well contracted after delivery D. A woman who has a first-degree laceration of the perineum

B. A woman who has type 1 diabetes and delivered a large-for-gestational-age infant via cesarean section

Which methods can all women use to reduce risk in the preconception period? (MA) A. Increasing saturated fats in their diet B. Abstaining from alcohol C. Taking a folic acid supplement D. Learning about inherited conditions that can affect pregnancy E. Achieving a healthy weight F. Waiting to update vaccines until after pregnancy to prevent exposure to teratogens

B. Abstaining from alcohol C. Taking a folic acid supplement D. Learning about inherited conditions that can affect pregnancy E. Achieving a healthy weight

During an emergency cesarean section, which interventions can help minimize maternal risk? A. Eliminating surgical instrument counts to expedite the procedure B. Administering antibiotics before skin incision C. Performing the Foley catheter insertion using clean technique to expedite the procedure D. Performing an interdisciplinary time-out before skin incision E. Having a prewarmed isolette available for the birth

B. Administering antibiotics before skin incision D. Performing an interdisciplinary time-out before skin incision

Which antihypertensive classifications are contraindicated in pregnancy? (MA) A. Beta blockers B. Angiotensin-converting enzyme (ACE) inhibitors C. Calcium channel blockers D. vasodilators E. Angiotensin receptor blockers (ARBs)

B. Angiotensin-converting enzyme (ACE) inhibitors E. Angiotensin receptor blockers (ARBs)

Which care would the nurse provide for an infant with macrosomia? (MA) A. Administer intravenous (IV) glucose. B. Assess for birth trauma. C. Monitor thermoregulation D. Educate families on proper handling. E. Prepare for mechanical ventilation.

B. Assess for birth trauma. C. Monitor thermoregulation D. Educate families on proper handling.

Which qualities are considered normal when assessing fetal heart rate (FHR)? (MA) A. FHR variability of <5 beats/min B. Baseline FHRof 140 beats/min C. FHR variability of 20 beats/min D. Baseline FHR of 100 beats/min E. Baseline FHR of 170 beats/min

B. Baseline FHRof 140 beats/min C. FHR variability of 20 beats/min

A woman at 37 weeks gestation calls and reports, "My water broke and I have bloody show. I am changing my pad every 10 minutes." Which advice would the nurse provide in response? A. Engage in activity such as walking. B. Call her health care provider and go to the hospital or birthing center. C. Advise the woman that overreacting is not good for her or the baby and to call back when she is in "true labor." D. Remain home, and only come in to the hospital or birthing center if contractions are regular, frequent, and intense.

B. Call her health care provider and go to the hospital or birthing center.

Which description is true of congenital cytomegalovirus (CMV)? A. Can lead to severe maternal anemia B. Can lead to neonatal hearing loss C. Often causes severe flulike symptoms in the mother D. Causes weeping skin lesions on the neonate

B. Can lead to neonatal hearing loss

A nurse is concerned that her pregnant patient is at risk for dehydration due to acute morning sickness. Which intervention is most appropriate in order to anticipate a treatment plan? A. Is the patient allergic to promethazine? B. Can the patient tolerate oral food and fluids? C. Does the patient have leukocytes in her urine? D. Does the patient have a patent intravenous (IV) site?

B. Can the patient tolerate oral food and fluids?

Which nursing interventions are appropriate for an infant experiencing withdrawal? (MA) A. Swaddle with the legs extended. B. Cluster care to minimize handling. C. Reduce environmental stimuli. D. Offer feedings every 3 to 4 hours. E. Offer nonnutritive sucking.

B. Cluster care to minimize handling. C. Reduce environmental stimuli. E. Offer nonnutritive sucking.

Which type of commercial formula requires dilution with equal parts water? A. Ready-to-feed B. Concentrate C. Powder D. Hydrolysate formula

B. Concentrate

Which cause of spontaneous abortion is the most common? A. Infection B. Congenital anomalies C. Vaginal Bleeding D. Anomalies of the maternal reproductive tract

B. Congenital anomalies

A nurse is beginning to care for a patient in labor. The health care provider has prescribed an intravenous (IV) infusion of oxytocin. The nurse makes sure that which implementations occur before initiation of the infusion? (MA) A. Placing the patient on complete bed rest B. Continuous electronic fetal monitoring C. An IV infusion of antibiotics D. Making sure that a vial of terbutaline is immediately available E. Preparing an IV infusion pump F. Placing a code cart at the patient's bedside

B. Continuous electronic fetal monitoring D. Making sure that a vial of terbutaline is immediately available E. Preparing an IV infusion pump

Which assessment findings in the laboring patient would suggest to the nurse that delivery is imminent? (MA) A. The woman can talk through contractions. B. Contractions are 1.5 to 2 minutes apart. C. Regular contractions are mild and 7 minutes apart. D. Mucous vaginal discharge is bloody. E. The woman is experiencing vomiting, nausea, and sweating. F. The patient complains of rectal pressure.

B. Contractions are 1.5 to 2 minutes apart. D. Mucous vaginal discharge is bloody. E. The woman is experiencing vomiting, nausea, and sweating. F. The patient complains of rectal pressure.

A G2/P1 woman presents for evaluation. She is excited, can talk through regular contractions every 7 to 10 minutes, and states that her pain is "manageable." She does not desire epidural anesthesia in labor. The nurse suspects this patient is in which phase of labor? A. Transitional labor B. Early labor C. The second stage of labor D. Active labor

B. Early labor

A patient in labor experiences minimal relief after nonpharmacologic interventions, and she is not coping well in labor. Which action would the nurse take next? A. Suggest an epidural. B. Educate on the risks and benefits of pharmacologic interventions. C. Encourage the patient to ambulate in the room. D. Administer the prescribed dose of intravenous pain medication.

B. Educate on the risks and benefits of pharmacologic interventions.

Based on the nurse's understanding of normal breast changes in pregnancy, which response would a nurse provide to a pregnant patient who complains of "tumors" and "cancer" in her breasts? A. Explain that breast cancer is often triggered by pregnancy. B. Explain that nodular breast tissue is normal during pregnancy. C. Tell the woman she is exhibiting signs of altered coping related to the pregnancy. D. Inform the woman that nodular breast tissue is abnormal during pregnancy, so she needs a mammogram.

B. Explain that nodular breast tissue is normal during pregnancy.

A patient reports frustration at the health care provider's decision not to perform all the diagnostic tests she wishes to have. Which types of patient education are appropriate? (MA) A. Tell the patient that her concerns will be reported to the health care provider. B. Explain to the patient that some abnormalities can be detected by more than one test. C. Inform the patient that she has the right to seek treatment elsewhere if she is not satisfied. D. Explain to the patient that even if all tests were normal, it would not rule out every abnormality. E. Inform the patient that each procedure carries risks to both patient and infant, and that risk increases when multiple procedures are performed.

B. Explain to the patient that some abnormalities can be detected by more than one test. D. Explain to the patient that even if all tests were normal, it would not rule out every abnormality. E. Inform the patient that each procedure carries risks to both patient and infant, and that risk increases when multiple procedures are performed.

Which action would the nurse take if a pregnant patient opts out of prenatal human immunodeficiency virus (HIV) testing? A. Report the mother to child protective services. B. Inform the mother that the baby will likely be screened for HIV after birth. C. Collect the sample anyway D. Request that the patient's sexual partners come in for screening.

B. Inform the mother that the baby will likely be screened for HIV after birth.

Which interventions would the nurse implement when caring for a newborn postcircumcision? (MA) A. Carefully remove yellow crust over glans. B. Loosely attach diaper. C. Clean glans with warm water and soap. D. Carefully document urinary output. E. Monitor for bleeding and signs of infection

B. Loosely attach diaper. D. Carefully document urinary output. E. Monitor for bleeding and signs of infection

During the fourth stage of labor, the nurse notes an increased amount of bleeding. The uterine fundus consistency is boggy. Which is the most appropriate initial nursing action? A. Alert the health care provider that the patient is experiencing postpartum hemorrhage. B. Massage the uterine fundus and observe for change in consistency. C. Administer uterotonic medications to increase uterine tone. D. Do nothing, as this is a normal finding in the fourth stage of labor and the tone is transient.

B. Massage the uterine fundus and observe for change in consistency.

A pregnant woman presents with clumpy, white vaginal discharge, vulvar pruritus, and burning. Which treatment would the nurse expect for this patient? A. Clindamycin B. Miconazole C. Metronidazole D. Imiquimod

B. Miconazole

Which interventions are most appropriate when caring for a patient admitted for preeclampsia on magnesium sulfate therapy? (MA) A. Encourage increased PO fluid intake. B. Monitor strict intake and output C. Place protective padding on the bed's side rails. D. Assist patient with ambulating to the bathroom instead of using a bedpan. E. Ensure that calcium gluconate is immediately available.

B. Monitor strict intake and output C. Place protective padding on the bed's side rails. E. Ensure that calcium gluconate is immediately available.

Which maternal response to pregnancy is consistent with the mother washing and folding baby clothes and cleaning the home? A. Anxiety B. Nesting C. Narcissism D. Emotional lability

B. Nesting

Which patient condition is a common breastfeeding complication in the first 24 hours following delivery? A. Mastitis B. Nipple soreness C. plugged milk ducts D. Everted nipples

B. Nipple soreness

Which nursing findings are concerning when assessing the breasts and nipples of a postpartum woman? (MA) A. Nipples are pink with intact skin. B. Nipples are pink with a blister line. C. Breasts are symmetric. D. Breasts are red and firm. E. Breasts are soft and nontender.

B. Nipples are pink with a blister line. D. Breasts are red and firm.

Which probabilities are associated with a man with hemophilia A having a child with a woman who does not have the disease and who is not a carrier? (MA) A. All of his sons will be affected. B. None of his sons will be affected. C. All of his daughters will be carriers. D. None of his daughters will be carriers.

B. None of his sons will be affected. C. All of his daughters will be carriers.

Which nursing comfort measures are recommended for a woman in her seventh month of pregnancy complaining of nasal congestion and occasional epistaxis? A. Warm compresses over the nasal area B. Normal saline nose spray C. Warm-mist humidifier in the room where she sleeps D. Oxymetazoline nasal spray

B. Normal saline nose spray

A nurse who is called to a patient's room notes that the patient's cesarean incision has separated. Which action is the highest priority for the nurse to perform? A. Flush the wound with normal saline. B. Notify the health care provider. C. Elevate the head of the patient's bed to a high Fowler's position. D. Flex the patient's knees.

B. Notify the health care provider.

A multiparous woman is admitted to the postpartum unit after a rapid labor and birth of a 4000-g infant. Her fundus is boggy, lochia is heavy, and vital signs are unchanged. The nurse has the woman void and massages her fundus, but the patient's fundus remains difficult to find and the lochia remains bright red and heavy. Which action would the nurse take next? A. Vigorously massage the fundus B. Notify the healthcare provider C. Recheck the vital signs D. Insert an indwelling catheter

B. Notify the healthcare provider

When evaluating a patient with suspected preterm premature rupture of membranes and preterm labor, the nurse recognizes which cues as signs of preterm labor? (MA) A. Dysuria and urinary frequency B. Pain and discomfort in the upper inner thighs C. Intermittent or constant lower back pain D. A sensation that the fetus is frequently "balling up" E. The perception of decreased fetal movements F. Diarrhea

B. Pain and discomfort in the upper inner thighs C. Intermittent or constant lower back pain D. A sensation that the fetus is frequently "balling up" F. Diarrhea

The mean arterial pressure (MAP) for a pregnant woman whose blood pressure is 106/70 is __

82

A postpartum patient is diagnosed with a pulmonary embolism and is receiving a heparin drip. The prescription reads as follows: 20,000 units heparin/500 mL of D5W infusing at 22 mL/h. The nurse would administer __ units of heparin.

880

Which statement by the nurse is the most supportive of the mother during the taking-hold phase? A. "Because I changed the last diaper, let me talk you through changing this diaper." B. "You are tired; I can take care of the baby for you." C. "When you are done feeding the baby, I will burp the baby for you." D. "If you are tired, just have your mother give the infant a bottle instead of breastfeeding."

A. "Because I changed the last diaper, let me talk you through changing this diaper."

When cardiovascular disease is severe during pregnancy, which health care providers may be a part of the interdisciplinary care team? (MA) A. Anesthesiologists B. Nurses C. Dermatologists D. Neonatologists E. Obstetricians F. Maternal-fetal medicine specialists

A. Anesthesiologists B. Nurses D. Neonatologists E. Obstetricians F. Maternal-fetal medicine specialists

Which intervention by the health care team will decrease the risk of infection for a patient with preterm premature rupture of membranes during labor, delivery, and the postpartum period? A. Avoiding frequent vaginal examinations to check for cervical dilation B. Inserting an indwelling urinary catheter to measure urinary output C. Checking maternal temperature every 12 hours D. Using clean technique when performing speculum examinations

A. Avoiding frequent vaginal examinations to check for cervical dilation

Which steps would the nurse take to facilitate effective discussion with a 15-year-old girl who is curious about methods of contraception but is hesitant to ask questions? (MA) A. Demonstrate proper contraceptive use to the adolescent. B. Reassure the adolescent that the discussion is confidential. C. Provide the adolescent with written material about different methods of contraception. D. Ask the adolescent about what she has learned about contraception from her peers. E. Inform the adolescent that a discussion about contraception cannot occur without a parent present.

A. Demonstrate proper contraceptive use to the adolescent. B. Reassure the adolescent that the discussion is confidential. C. Provide the adolescent with written material about different methods of contraception. D. Ask the adolescent about what she has learned about contraception from her peers.

Which disorders have a known chromosomal inheritance pattern? (MA) A. Down Syndrome B. Turner Syndrome C. Marfan Syndrome D. Fragile X Syndrome E. Edwards Syndrome

A. Down Syndrome B. Turner Syndrome E. Edwards Syndrome

Which preventive intervention would the nurse implement for a newborn with numerous hyperbilirubinemia risk factors? A. Encourage early feeding. B. Collect total serum bilirubin (TSB). C. Initiate phototherapy. D. Prepare for exchange transfusion.

A. Encourage early feeding.

Which approaches would the nurse suggest to new parents to promote adaptation? (MA) A. Encourage the parents to delay visitors. B. Emphasize the importance of a strict schedule. C. Encourage the parents to accomplish household tasks when the baby sleeps. D. Encourage the parents to enlist family and friends to help with household tasks. E. Emphasize that the priority during the first 4 to 6 weeks should be caring for themselves and the baby.

A. Encourage the parents to delay visitors. D. Encourage the parents to enlist family and friends to help with household tasks. E. Emphasize that the priority during the first 4 to 6 weeks should be caring for themselves and the baby.

A G4/P3 patient experiencing precipitate labor presents to the labor wing fully dilated and at +1 station stating that she feels a strong, involuntary urge to push. Which immediate intervention would the nurse take? A. Encourage the patient to push in a side-lying position. B. Initiate an oxytocin infusion to prevent postpartum hemorrhage. C. Assess for umbilical cord prolapse. D. Allow the fetus to rest and descend until birth is imminent.

A. Encourage the patient to push in a side-lying position.

A woman with an epidural is 10 cm dilated and the baby is at 0 station. There is a reassuring fetal heart pattern, although the woman does not feel the urge to push. Which are appropriate actions by the nurse? (MA) A. Encourage the patient to wait to push until she feels the urge. B. Perform another vaginal examination to check for dilation. C. Encourage the patient to push so that the baby can be born. D. Encourage frequent position changes while remaining in the bed. E. Turn off the epidural so the patient can feel the urge to push.

A. Encourage the patient to wait to push until she feels the urge. D. Encourage frequent position changes while remaining in the bed.

Which nursing education is appropriate to provide to a pregnant patient reporting hyperemia? A. Encourage the woman to brush her teeth carefully. B. Advise the woman to have her blood pressure checked regularly. C. Encourage the woman to wear supportive hosiery. D. Advise the woman not to lie flat on her back.

A. Encourage the woman to brush her teeth carefully.

Which response to pregnancy is expected from a 2-year-old child? A. Jealousy B. Depression C. Fear of role change D. Nesting

A. Jealousy

Which factors are associated with the development of complications in infants of mothers with diabetes? A. Metabolic control before conception B. Gestational age of newborn C. Severity of mother's diabetes D. Age of diabetes onset E. Degree of maternal neurologic complications

A. Metabolic control before conception C. Severity of mother's diabetes D. Age of diabetes onset

The nurse is caring for a woman in the second stage of labor whose contractions have become more intense. What actions could the nurse take to assist this woman? (MA) A. Model modified-paced breathing. B. Model slow-paced breathing. C. Monitor for hyperventilation. D. Conduct acupuncture. E. Ask the patient if it is OK to hold her hand.

A. Model modified-paced breathing. C. Monitor for hyperventilation. E. Ask the patient if it is OK to hold her hand.

Which nursing intervention would be included in the care of a woman with lupus during her first trimester of pregnancy? A. Monitor for liver function. B. Monitor for increased joint pain and fatigue. C. Provide patient education on remission of lupus. D. Administer subcutaneous heparin for thrombosis prevention.

A. Monitor for liver function.

A patient presents with bleeding caused by placenta previa in late pregnancy. Which physical assessments are the most important for the nurse to perform? (MA) A. Monitor urinary output. B. Monitor vaginal bleeding C. Assess vital signs for the patient. D. Monitor fetal heart rate and contraction patterns. E. Perform digital vaginal examination for dilation. F. Measure fundal height.

A. Monitor urinary output. B. Monitor vaginal bleeding C. Assess vital signs for the patient. D. Monitor fetal heart rate and contraction patterns.

Which classes are appropriate for a 34-week pregnant patient and her partner who report being nervous about the upcoming delivery and baby care? (MA) A. Newborn care B. Prenatal exercise C. Early prenatal care D. Preconception care E. Childbirth preparation

A. Newborn care E. Childbirth preparation

Which drugs would the nurse anticipate administering for tocolysis during preterm labor? (MA) A. Nifedipine B. Indomethacin C. Magnesium sulfate D. Lebetalol E. Betamethasone

A. Nifedipine B. Indomethacin

The nurse is providing care for a patient in labor, and the health care provider has just stated the patient's need for a forceps-assisted delivery. Which actions would the nurse's preparation include? (MA) A. Obtaining a urinary catheter B. Establishing intravenous (IV) access C. Performing a head-to-toe assessment D. Educating the patient about the risk for lacerations E. Preparing the forceps using aseptic technique F. Monitoring the fetal heart rate for signs of distress

A. Obtaining a urinary catheter B. Establishing intravenous (IV) access E. Preparing the forceps using aseptic technique F. Monitoring the fetal heart rate for signs of distress

In the fourth stage of labor, the nurse monitors the hemodynamic stability of the postpartum patient through which assessment? A. Obtaining maternal vital signs every 15 minutes in the first postpartum hour B. Assessing the couplet for bonding C. Obtaining maternal vital signs every 30 minutes in the first postpartum hour D. Assessing the color, amount, and odor of urine after the first void

A. Obtaining maternal vital signs every 15 minutes in the first postpartum hour

At which time would it be most effective for the nurse to educate a new mother about circumcision care? A. On the second postpartum day B. With discharge teaching on the day of discharge C. During admission while going over the consent forms D. Within the first 24 hours after birth

A. On the second postpartum day

While awaiting the delivery of the placenta in the third stage of labor, the nurse observes a large gush of blood and an increase in the length of the umbilical cord. Which event would the nurse recognize has probably occurred? A. Placental separation from the uterine wall B. Placental abruption C. Umbilical cord detachment from the placenta D. Postpartum hemorrhage as a result of retained placenta

A. Placental separation from the uterine wall

A child is born with a genetic disorder and is found to have 46 chromosomes. Which type of disorder could have caused the genetic disorder? (MA) A. Polygenic Disorder B. Single Gene Disorder C. Chromosomal Disorder D. Autosomal Recessive Disorder E. Autosomal Dominant Disorder

A. Polygenic Disorder B. Single Gene Disorder D. Autosomal Recessive Disorder E. Autosomal Dominant Disorder

Wound dehiscence is a sign of a wound infection and is characterized by which finding? A. Separation of the wound site B. Purulent drainage at the wound site C. Bleeding at the wound site D. Edema at the wound site

A. Separation of the wound site

Which sign would lead the nurse to suspect ectopic pregnancy in a patient with a missed period? A. Severe, localized abdominal pain in the left lower abdominal quadrant B. Vaginal bleeding after intercourse C. Nausea and vomiting D. Painless, bright-red vaginal bleeding

A. Severe, localized abdominal pain in the left lower abdominal quadrant

Which type of anemia is an inherited anemia? A. Sickle cell anemia B. aplastic anemia C. Iron deficiency anemia D. Pernicious anemia

A. Sickle cell anemia

Which infant actions are common cues indicating a readiness to feed? (MA) A. Sucking B. Rooting C. Mouthing D. Crying E. Hand-to-mouth movements F. Vocalizations

A. Sucking B. Rooting C. Mouthing E. Hand-to-mouth movements

Which factors are likely to support breastfeeding as the family's feeding method choice? (MA) A. Support from family B. Formula access C. Cultural practices D. Return to work E. Healthcare provider knowledge

A. Support from family C. Cultural practices E. Healthcare provider knowledge

Which phrases describe the goals of perinatal education classes? (MA) A. To decrease anxiety related to childbirth B. To teach parents how to perform cardiopulmonary resuscitation (CPR) for their children C. To provide knowledge related to the labor process and birth D. To increase knowledge of comfort relief measures during labor E. To teach the expectant families how to parent and discipline their children

A. To decrease anxiety related to childbirth C. To provide knowledge related to the labor process and birth D. To increase knowledge of comfort relief measures during labor

Which passenger position would interfere with a safe vaginal birth? A. Transverse fetal lie B. Flexed fetal head C. Cephalic presentation D. Longitudinal fetal lie

A. Transverse fetal lie

Which contraception methods would be appropriate for a woman who is intolerant to hormonal therapy? A. Tubal ligation B. Emergency contraception (EC) C. Contraceptive vaginal ring D. Diaphragm with spermicide E. Copper intrauterine device (IUD)

A. Tubal ligation D. Diaphragm with spermicide E. Copper intrauterine device (IUD)

Which two factors trigger cardiovascular adaptation after birth? (MA) A. Umbilical cord clamping B. Tactile stimulation C. Chest compression D. Initiation of respiration E. Chemical factors

A. Umbilical cord clamping D. Initiation of respiration

Which characteristic describes sleep disturbances associated with postpartum depression? A. Unrestful sleep B. Caused by the baby's presence C. Excessive sleeping D. Attributed to the need for nighttime feedings

A. Unrestful sleep

Which factors may affect the sperm quality in a male? (MA) A. Untreated gonorrhea B. Nightly hot baths C. Impotence D. Varicocele E. Obesity

A. Untreated gonorrhea B. Nightly hot baths E. Obesity

When witnessing informed consent for cesarean delivery, which risks to the patient is the nurse acknowledging the patient understands? (MA) A. Urinary tract infection or trauma to the bladder B. Cervical laceration or trauma C. Increased risk for blood clots, which can be in the legs or can travel to the heart or lungs D. Injury to the newborn including lacerations, bruising, or fractures E. The need for cesarean birth with every delivery after this procedure regardless of the incision type F. The risk that the baby's lungs might be compromised or need extra support at birth

A. Urinary tract infection or trauma to the bladder C. Increased risk for blood clots, which can be in the legs or can travel to the heart or lungs D. Injury to the newborn including lacerations, bruising, or fractures F. The risk that the baby's lungs might be compromised or need extra support at birth

A woman, 1 day postpartum, is being carefully monitored after a significant postpartum hemorrhage (PPH). Which finding would the nurse report to the health care provider? A. Urine output of 160 mL for the past 8 hours B. Weight loss of 2lbs after delivery C. Pulse rate of 68 beats/min D. Fundus firm at umbilicus

A. Urine output of 160 mL for the past 8 hours

Which factor explains the cause of hot flashes during menopause? A. Vasomotor instability B. Increased estrogen production C. Progesterone secretion D. Changes to the hypothalamus

A. Vasomotor instability

Which statement by a woman with multiple sexual partners after a tubal ligation would indicate that additional teaching regarding preventive screening is necessary? A. "I will come yearly for a Papanicolaou (Pap) test." B. "I don't have to worry about using condoms for sexually transmitted infections (STIs)." C. "I am continuing to do monthly vulvar self-examinations." D. "I don't have to get a colonoscopy."

B. "I don't have to worry about using condoms for sexually transmitted infections (STIs)."

A patient who is 10 weeks pregnant feels uncomfortable having a transvaginal ultrasound and asks the nurse if there is any way she could have a transabdominal ultrasound instead. Which responses from the nurse are appropriate? (MA) A. "We can perform a transabdominal ultrasound if your bladder is full." B. "If it makes you feel more comfortable, you can insert the probe yourself." C. "We would not be able to perform a transabdominal ultrasound until next week." D. "I will have the health care provider change the order to a transabdominal ultrasound." E. "Because you are in your first trimester, the uterus and embryo are difficult to visualize with a transabdominal ultrasound."

B. "If it makes you feel more comfortable, you can insert the probe yourself." E. "Because you are in your first trimester, the uterus and embryo are difficult to visualize with a transabdominal ultrasound."

Which nursing information is appropriate when teaching a postpartum woman about how to increase perineal muscle tone? A. "Ambulate early and frequently." B. "Perform Kegel exercises 30 times per day." C. "Perform pelvic lifts several times per day." D. "Tighten and relax the abdominal muscles several times per day."

B. "Perform Kegel exercises 30 times per day."

A 47-year-old woman presents at a clinic complaining of irregular periods and states, "I am too young for menopause." Which is the best response from the nurse? A. "Yes, menopause occurs only after age 50." B. "Some women go through menopause earlier than others." C. "The health care provider will do a test to determine the cause." D. "Yes, menopause occurs only after age 60."

B. "Some women go through menopause earlier than others."

Which educational information related to activity would the nurse provide to a postpartum woman who delivered by cesarean birth? (MA) A. "Housework may be resumed within 2 weeks." B. "Stair-climbing should be limited for the first 2 weeks." C. "Leg muscles should be used to lift rather than the abdominal muscles." D. "Walking and swimming are encouraged to prevent blood clot formation." E. "Abdominal exercises should be done several times per day after discharge to increase muscle tone."

B. "Stair-climbing should be limited for the first 2 weeks." C. "Leg muscles should be used to lift rather than the abdominal muscles."

Which explanation of cesarean delivery incisions would a nurse provide to a patient? A. "There are a total of three incisions that will be made in your body by the surgeon." B. "The surgeon may make a vertical incision or a transverse incision on your skin." C. "You will not be able to the see the incision once it is fully healed if the surgeon makes a Pfannenstiel incision on your skin." D. "If you have a transverse incision on your uterus, you can never have a vaginal birth."

B. "The surgeon may make a vertical incision or a transverse incision on your skin."

Which nursing response is appropriate for a 20-week gestation woman who is concerned about gaining 12 lb (5.4 kg) in her pregnancy? A. "I will tell the health care provider that you are worried. The health care provider will tell you what to do." B. "Your weight gain is about average for this point in your pregnancy. What concerns you about it?" C. "You really have gained a lot! I will consult with the nutritionist for you." D. "A lot of your weight gain is probably fluid. I suggest you decrease your fluid intake and increase your salt intake."

B. "Your weight gain is about average for this point in your pregnancy. What concerns you about it?"

The nurse is instructing a childbirth class on abdominal breathing and tells a patient that her baseline respiratory rate is 22 breaths/min. What should the patient's rate be while performing slow-paced breathing techniques? A. 9 breaths/min B. 11 breaths/min C. 15 breaths/min D. 20 breaths/min

B. 11 breaths/min

A postpartum patient who had a spinal block for a scheduled cesarean delivery complains of a headache. Which intervention would the nurse anticipate? A. Oral (PO) administration of promethazine B. Administration of blood patch by the anesthesia provider C. Having patient sit up in chair to help relieve headache D. Allowing the spinal headache to run its course

B. Administration of blood patch by the anesthesia provider

A woman is 10 cm dilated and feels occasional rectal pressure at the peak of a contraction, and the fetal station is 0. There is a reassuring fetal heart tracing. Which nursing action is appropriate at this time? A. Recheck the cervical dilation to ensure she is 10 cm. B. Allow the woman to labor down for passive descent. C. Encourage the woman to push with every contraction. D. Prepare for cesarean delivery for cephalopelvic disproportion.

B. Allow the woman to labor down for passive descent.

The nurse assesses the blood pressure of a pregnant patient with preeclampsia and notes that is elevated. Which antihypertensive classification would be avoided for this patient? A. Calcium channel blockers B. Angiotensin-converting enzyme (ACE) inhibitors C. Vasodilators D. Beta blockers

B. Angiotensin-converting enzyme (ACE) inhibitors

Which nursing intervention is appropriate to help prevent thrombophlebitis in a postpartum woman who delivered by cesarean birth? A. Administer warfarin therapy by mouth. B. Encourage early and frequent ambulation. C. Administer intravenous (IV) furosemide to increase urinary output. D. Encourage increased fluid intake.

B. Encourage early and frequent ambulation.

Which nursing interventions can help prevent abdominal distention and gas discomfort for a postpartum woman who delivered by cesarean birth? (MA) A. Administer oral (PO) narcotics. B. Encourage increased ambulation. C. Encourage increased oral (PO) fluid intake. D. Discourage the use of drinking straws. E. Encourage intake of carbonated beverages.

B. Encourage increased ambulation. C. Encourage increased oral (PO) fluid intake. D. Discourage the use of drinking straws.

Which solution can the nurse consider to encourage a positive birth experience for the patient and her support person following delivery? A. Offer to take the baby to the nursery to promote rest for the patient and the support person. B. Encourage the family to talk about the labor and delivery experience. C. Immediately after birth, remove amniotic fluid and vernix from the baby before placing it skin-to-skin with the mother. D. Provide all infants with pacifiers to promote infant and family rest.

B. Encourage the family to talk about the labor and delivery experience.

A nurse caring for a patient experiencing maternal exhaustion who desires a natural labor would implement which intervention to promote normal labor progress and decrease fatigue? A. Limit intravenous fluids (IV) to prevent overhydration. B. Encourage the patient to take a warm shower or bath. C. Insert an indwelling urinary catheter so the patient does not have to ambulate as frequently. D. Restrict visitation to given time intervals to allow for sleep.

B. Encourage the patient to take a warm shower or bath.

Which method would the nurse recognize as a low-tech assessment of fetal well-being? A. Fetal aneuploidy screening B. Fetal kick counts C. Ultrsonography D. Biophysical profile assessment

B. Fetal kick counts

A nurse is caring for the following labor patients. Which patients would the nurse be prepared to monitor closely for signs of postpartum hemorrhage (PPH)? (MA) A. G1 P0000, delivered a 29-week fetal demise vaginally after 8 hours in labor B. G2 P1001, delivered a 4200-g neonate vaginally after 4 hours of labor C. G2 P0010, delivered a 3750-g neonate by cesarean section for breech presentation D. G4 P3003, delivered a 3500-g neonate by cesarean section with a placenta accreta E. G3 P0200, delivered a 3900-g neonate vaginally after 36 hours in labor

B. G2 P1001, delivered a 4200-g neonate vaginally after 4 hours of labor D. G4 P3003, delivered a 3500-g neonate by cesarean section with a placenta accreta E. G3 P0200, delivered a 3900-g neonate vaginally after 36 hours in labor

The nurse midwife writes in the progress note that the baby is in the occiput posterior presentation. The nurse knows that which maternal position will help facilitate rotation of the fetal head? A. Side-lying B. Hands and knees C. Semi-recumbent D. Lithotomy

B. Hands and knees

Which assessment findings suggest excessive blood loss requiring immediate intervention for a postpartum patient who had a cesarean delivery? (MA) A. Firm fundus, midline B. Heart rate of 120 beats/min C. Blood pressure of 80/40 mm Hg D. Urinary output of 20 mL/hour E. Abdominal distension and severe pain

B. Heart rate of 120 beats/min C. Blood pressure of 80/40 mm Hg D. Urinary output of 20 mL/hour E. Abdominal distension and severe pain

Which nursing action is appropriate when a 37-week pregnant woman complains of "getting lightheaded and dizzy" while having her fetal heart tones checked? A. Assist the woman to sit up. B. Help the patient turn to her left side. C. Check the woman's blood pressure. D. Notify the health care provider.

B. Help the patient turn to her left side.

Which TORCH infections may be transmitted perinatally? (MA) A. Toxoplasmosis B. Herpes simplex virus C. Gonorrhea D. Varicella zoster E. Rubella F. Cytomegalovirus

B. Herpes simplex virus C. Gonorrhea D. Varicella zoster F. Cytomegalovirus

Which risk factor identified by the nurse places the patient at risk for preterm labor (PTL)? A. Family history of cervical cancer B. Homelessness C. Hypertension D. History of birth of a child at 37 weeks

B. Homelessness

Which physiologic finding influences balance changes in a pregnant woman? A. Kyphosis B. Lordosis C. Diastasis recti D. Back pain

B. Lordosis

A patient is a G2/P0 at 32 weeks pregnant and experiencing regular contractions. She has a multifetal gestation pregnancy, a history of preterm birth, and has had recurrent bacterial vaginosis throughout the pregnancy. Her BMI is 22, she is 30 years' old, and she is a former smoker who quit two years ago. Of the data provided, which are risk factors for preterm labor? (MA) A. Age B. Multifetal gestation C. Recurrent bacterial vaginosis D. Former smoker status E. BMI 22

B. Multifetal gestation C. Recurrent bacterial vaginosis

A woman received 25 mg of meperidine intravenously 1 hour before delivery. Which drug would the nurse have readily available? A. Ibuprofen B. Naloxone C. Nalbuphine D. Pitocin

B. Naloxone

Occiput posterior refers to which component of the birth process? A. Powers B. Passenger C. Passage D. Psyche

B. Passenger

Which pregnancy-related condition is related to both maternal and fetal influences? A. Amniotic banding B. Polyhydramnios C. Nuchal cord D. Preeclampsia

B. Polyhydramnios

The nurse notes recurrent late decelerations that do not respond to intrauterine resuscitation on the fetal monitor attached to a patient with a concealed placental abruption. Which order would the nurse anticipate? A. Administer pain medication as prescribed. B. Prepare for emergency cesarean birth. C. Turn the patient to the left lateral position. D. Palpate the fundal tone.

B. Prepare for emergency cesarean birth.

Which term best describes the transition between childhood and sexual maturity? A. Menarche B. Puberty C. Menopause D. Menstruation

B. Puberty

In which areas of the body do the events of the menstrual cycle occur concurrently? A. The endometrium, hypothalamus, and ovaries B. The endometrium, hypothalamus, pituitary gland, and ovaries C. The endometrium, pituitary gland, and ovaries D. The endometrium, hypothalamus, gonads, and ovaries

B. The endometrium, hypothalamus, pituitary gland, and ovaries

In which situation is there an increased likelihood for prolonged labor? A. The woman is a teen mother. B. The woman is nulliparous. C. The woman has been diagnosed with an incompetent cervix. D. The woman has a history of postpartum hemorrhage.

B. The woman is nulliparous.

Which health risk is associated with perimenopause? A. Reproductive cancers B. Flushing C. Cardiovascular disease D. Vaginal dryness

C. Cardiovascular disease

Which type of placenta previa would the nurse recognize as being most dangerous? A. Concealed previa B. Marginal previa C. Complete previa D. Partial previa

C. Complete previa

During which phase of accepting the pregnancy does the father begin to negotiate his role as a parent? A. Announcement phase B. Moratorium phase C. Focusing phase D. Couvade phase

C. Focusing phase

For which TORCH infections would the nurse anticipate administering immune globulin? (MA) A. Toxoplasmosis B. Gonorrhea C. Hepatitis B virus D. Varicella zoster E. Syphilis F. Rubella

C. Hepatitis B virus D. Varicella zoster

Which associated risk would a fertility specialist discuss with a couple who is considering the use of assisted reproductive technology (ART)? Weight loss B. Polycystic ovary syndrome C. Ovarian hyperstimulation D. Tubal adhesions

C. Ovarian hyperstimulation

During fundal assessment 30 minutes after a cesarean delivery, the patient reports discomfort and exhaustion and asks the nurse, "Why do you keep poking at me?" Which response from the nurse is appropriate? A. "I'll wait until you're on the postpartum floor to assess you again." B. "I need to do this assessment to make sure your incision is healing correctly." C. "This assessment must be done every 30 minutes until you transfer out of the recovery room." D. "A cesarean delivery puts you at risk for excessive bleeding, so it's important to assess you frequently."

D. "A cesarean delivery puts you at risk for excessive bleeding, so it's important to assess you frequently."

A patient with gestational diabetes experiences rupture of membranes at 36 weeks gestation. Which patient teaching statement regarding this condition is accurate? A. "Rupture of membranes is more common in women with gestational diabetes because of the excess amounts of glucose in your urine." B. "Rupture of membranes at this time in your pregnancy is normal and sometimes happens before labor begins." C. "Your membranes likely ruptured because of excess ketones in your blood." D. "Preterm rupture of membranes is more common in women with gestational diabetes, and it might be related to overdistention of your uterus."

D. "Preterm rupture of membranes is more common in women with gestational diabetes, and it might be related to overdistention of your uterus."

Which educational information regarding breast care would the nurse provide for a mother who chooses to bottle-feed her infant? (MA) A. "Apply a hot compress to the breasts for comfort." B. "Pump the breasts if they become engorged after birth." C. "Clean breasts with hot water and soap in the shower." D. "Take analgesics as needed for engorgement discomfort." E. "Apply ice packs to the breasts for several days."

D. "Take analgesics as needed for engorgement discomfort." E. "Apply ice packs to the breasts for several days."

The nurse recognizes that a steady trickle of bright red blood from the vagina in the presence of a firm fundus may indicate which condition? A. Uterine atony B. Retained placenta C. Infection of the uterus D. A laceration within the genital tract

D. A laceration within the genital tract

Which patient would most likely require general anesthesia during labor and delivery? A. A patient with a vaginal infection B. A patient with severe pain from uterine contractions C. A patient requiring a vaginal episiotomy D. A patient refusing regional and local anesthetic methods who needs a cesarean delivery

D. A patient refusing regional and local anesthetic methods who needs a cesarean delivery

Maternity nurses often have to answer questions about the many, sometimes unusual, ways people have tried to make the birthing experience more comfortable. Which information would the nurse consider when educating patients and support people about nonpharmacologic pain management techniques? A. Music supplied by the support person has to be discouraged because it could disturb others or upset the hospital routine. B. Women in labor can benefit from sitting in a bathtub, but they must limit immersion to no longer than 15 minutes at a time. C. Effleurage is permissible, but counterpressure is almost always counterproductive. D. Acupressure to either side of the spine can facilitate the release of endorphins and decrease some back pain.

D. Acupressure to either side of the spine can facilitate the release of endorphins and decrease some back pain.

Which location for a superficial venous thrombosis is the most common? A. Top of the foot B. Across the knees C. Groin area D. Behind the knees

D. Behind the knees

Which physiologic change in a pregnant woman is likely responsible for frequent vaginal yeast infections? A. Increased estrogen levels B. Increased prolactin levels C. Decreased progesterone levels D. Increased glycogen secretion within the vaginal walls

D. Increased glycogen secretion within the vaginal walls

After administration of oral (PO) labetalol for a patient with gestational hypertension, the nurse notes a blood pressure (BP) of 178/96 mm Hg. The patient also reports headache and blurry vision. Which order from the patient's health care provider would the nurse anticipate? A. Up ad lib B. Lisinopril C. Salt-free diet D. Magnesium sulfate

D. Magnesium sulfate

A patient experiences nausea and vomiting during labor. Which medication might the nurse anticipate administering? A. Fentanyl B. Naloxone C. Nalbuphine D. Promethazine

D. Promethazine

Match the fetal surveillance method to its corresponding advantage. Noninvasive and suitable for most patients in labor

External electronic fetal monitoring

Match the fetal surveillance method to its corresponding advantage. Accurate measurement of fetal heart rate

Fetal scalp electrode

Match the expected event with the stage of labor. Cervical effacement and dilatation from onset of true labor to complete dilatation

First stage

Match the expected event with the stage of labor. Expulsion of the placenta to physical recovery of mother and infant

Fourth stage

Match the fetal surveillance method to its corresponding advantage. Accurate measurement of uterine contractions, including intensity

Intrauterine pressure catheter

What is the method of heat loss with the nursing action that would prevent it? Move the crib away from air vents.

convection

The nurse understands that an early postpartum hemorrhage is defined as an estimated blood loss greater than __ mL in the first 24 hours after a vaginal delivery.

500

Which statements explain how plasma volume returns to baseline after delivery? (MA) A. Increased oxytocin secretion accelerates fluid depletion. B. Profuse sweating aids in decreasing plasma volume levels. C. Sodium retention aids in the diaphoresis of excess plasma volume. D. Increased urinary output promotes the excretion of excess plasma volume. E. Decreased aldosterone hormone levels promote diuresis of excess plasma volume.

B. Profuse sweating aids in decreasing plasma volume levels. D. Increased urinary output promotes the excretion of excess plasma volume. E. Decreased aldosterone hormone levels promote diuresis of excess plasma volume.

The nurse is caring for a patient who had a forceps delivery that caused a perineal hematoma. Which nursing intervention is most appropriate? A. Administer topical analgesic ointment as prescribed. B. Provide the patient with an ice pack and educate her about its use. C. Encourage the patient to lie on her side as much as possible until the injury heals. D. Educate the patient about the fact that the hematoma was caused by the introduction of forceps.

B. Provide the patient with an ice pack and educate her about its use.

The nurse caring for a pregnant patient with gestational diabetes at 33 weeks gestation assesses the fundal height at 36 cm. Which actions by the nurse are correct regarding this finding? (MA) A. Assure the patient that the fetus is growing appropriately for its age. B. Recommend that the health care provider order an ultrasound. C. Educate the patient on the importance of limiting weight gain for the remainder of the pregnancy. D. Perform a fetal nonstress test and report the findings to the health care provider. E. Assist the patient with scheduling a cesarean delivery at 37 weeks.

B. Recommend that the health care provider order an ultrasound. D. Perform a fetal nonstress test and report the findings to the health care provider.

For which reasons would patient education focus on smoking cessation during pregnancy? (MA) A. Risk for low blood sugar B. Risk for premature birth C. Risk for congenital anomalies D. Risk for increased fetal circulation E. Risk for low birth weight

B. Risk for premature birth E. Risk for low birth weight

Maternal role attainment is a process that spans from pregnancy through which time period? A. Immediately after birth, when the mother can practice her newborn care skills B. Several months after delivery C. Until her first pediatrician visit postdischarge D. After 1 year postpartum

B. Several months after delivery

The nurse understands that which conditions are risks for a uterine inversion? (MA) A. Precipitous delivery B. Short umbilical cord C. Polyhydramnios D. Fundal implantation of the placenta E. Prolonged labor

B. Short umbilical cord D. Fundal implantation of the placenta E. Prolonged labor

An 18-year-old presents with a history of hip pain and is found to have avascular necrosis. Which genetic disorder may be present? A. Cystic Fibrosis B. Sickle cell disease C. Tay-Sachs Disease D. Edwards Disease

B. Sickle cell disease

Which genetic disorders can be understood using Punnett squares? (MA) A. Polygenic Disorder B. Single Gene Disorder C. Chromosomal Disorder D. Autosomal Recessive Disorder E. Autosomal Dominant Disorder

B. Single Gene Disorder D. Autosomal Recessive Disorder E. Autosomal Dominant Disorder

A woman is seeking a long-term contraceptive method that does not require taking daily medication or applying a device before intercourse. Which contraceptive methods would the nurse recommend? (MA) A. Lea's Shield B. Single-rod etonogestrel implant C. Contraceptive sponge D. Intrauterine device (IUD) E. Combined oral contraceptives (COCs)

B. Single-rod etonogestrel implant D. Intrauterine device (IUD)

Which assessment data would support a diagnosis of pulmonary embolism (PE)? A. Calf pain with dorsiflexion of the foot B. Sudden onset of pleuritic chest pain and dyspnea C. Left-sided chest pain and diaphoresis D. Bilateral crackles and low-grade fever

B. Sudden onset of pleuritic chest pain and dyspnea

Which emotional expressions of "baby blues" are most common? A. Exhaustion and withdrawal B. Tearfulness and tiredness C. Anxiety and hopelessness D. Paranoia and insomnia

B. Tearfulness and tiredness

A patient requests nonhormonal intrauterine birth control. The nurse prepares which device for insertion by the nurse midwife? A. The medroxyprogesterone injection B. The Copper T380A intrauterine device C. The levonorgestrel intrauterine system D. The etonogestrel implant

B. The Copper T380A intrauterine device

Which physical change indicates impending ovulation? A. Necrotic endometrium B. Thin, stringy cervical mucus C. Thickened endometrial secretions D. Dilation of endometrial blood vessels

B. Thin, stringy cervical mucus

Which signs would the nurse assess in a newborn exposed to tobacco while in utero? (MA) A. Flat nasal bridge B. Tremors C. Signs of prematurity D. Open spinal cord defect E. Colic/fussy behavior F. A lower-than-expected weight

B. Tremors C. Signs of prematurity E. Colic/fussy behavior F. A lower-than-expected weight

Which chromosomal abnormalities result in a form of Down syndrome? A. Trisomy 18 B. Trisomy 21 C. Polysomy X D. Monosomy X E. Translocation

B. Trisomy 21 E. Translocation

A G5/P4 laboring patient with suspected fetal intrauterine growth restriction has just experienced spontaneous rupture of membranes. On examination, the nurse notes that the cervix is dilated 3 cm and is 70% effaced and that the fetal station is 0. Fetal heart tracing shows recurrent and severe variable decelerations. Which explanation is the most likely cause for this change in fetal heart rate? A. High fetal station B. Umbilical cord prolapse C. Fetal intrauterine growth restriction D. Grand multiparity

B. Umbilical cord prolapse

Which classic findings would the nurse identify in a patient with placental abruption (abruptio placentae)? (MA) A. Absent vaginal bleeding B. Uterine irritability with low-intensity contractions C. Severe abdominal cramping D. Painless vaginal bleeding E. Boardlike abdomen

B. Uterine irritability with low-intensity contractions C. Severe abdominal cramping E. Boardlike abdomen

Which interventions would the nurse implement for a breastfed newborn who has lost 8% of his or her birth weight? (MA) A. Recommend a feeding schedule of every 3 to 4 hours. B. Weigh the infant daily using same scale. C. Evaluate feeding technique and positioning. D. Demonstrate waking techniques for feeding a sleepy baby. E. Evaluate the newborn suck and latch.

B. Weigh the infant daily using same scale. C. Evaluate feeding technique and positioning. D. Demonstrate waking techniques for feeding a sleepy baby. E. Evaluate the newborn suck and latch.

Which statement indicates a need for further teaching regarding cesarean delivery? A. "I might need a blood transfusion after the surgery." B. "Having a C-section might make my hospital stay longer." C. "Because I am not having a vaginal birth, my babies can't be harmed during delivery." D. "Depending on how they do the incision, I may be able to have a vaginal delivery if I become pregnant again."

C. "Because I am not having a vaginal birth, my babies can't be harmed during delivery."

Which statement made by a patient having an amniocentesis indicates an understanding of the risks associated with the diagnostic test? A. "I will not need Rho(D) immune globulin after this even though I am Rh-negative." B. "There is a moderate risk I will get an infection as a result of this test." C. "Because the needle is guided by ultrasound, the risk for injury to the baby is small." D. "Because I am at 14 weeks gestation, there is a higher chance of the baby having a foot deformity after the procedure."

C. "Because the needle is guided by ultrasound, the risk for injury to the baby is small."

Which nursing education is appropriate for a pregnant patient when discussing breast changes? A. Increased relaxin levels may lead to darkening of the areola. B. Secretion of colostrum is normal in the first trimester. C. "Bumps" on the nipples are normal. D. Breasts lose sensitivity during pregnancy.

C. "Bumps" on the nipples are normal.

Which parental statement indicates a need for further sleep safety education? A. "My baby sleeps on her back." B. "I dress my baby in a sleep sack for sleep." C. "I cover my baby with a warm quilt at night." D. "My baby sleeps in a bassinet next to my bed."

C. "I cover my baby with a warm quilt at night."

Which parental statement would the nurse interpret as indicating a need for further education about newborn body processes? A. "My baby's breathing might be variable and irregular." B. "I will dress my baby similar to how I am dressed except with an additional blanket." C. "I will check my baby's temperature in his bottom when he is sick." D. "My baby should have at least six wet diapers a day."

C. "I will check my baby's temperature in his bottom when he is sick."

A nurse is reviewing discharge teaching with a patient who has a urinary tract infection (UTI). Which statements by the patient indicate understanding of the teaching? (MA) A. "I will perform perineal care and apply a perineal pad in a back-to-front direction." B. "I will drink cranberry and prune juices to make my urine more acidic." C. "I will drink large amounts of water to flush the bacteria from my urinary tract." D. "I will go back to breastfeeding after I have finished taking the antibiotic." E. "I will take analgesics for any discomfort."

C. "I will drink large amounts of water to flush the bacteria from my urinary tract." E. "I will take analgesics for any discomfort."

The nurse recognizes that the patient understands the teaching when she makes which statement? A. "Premonitory labor signs include rupture of membranes." B. "I will know I'm in labor when I lose my mucous plug." C. "Increasing clear vaginal secretions could mean labor will begin soon." D. "Bright red bleeding is considered bloody show and means labor will begin soon."

C. "Increasing clear vaginal secretions could mean labor will begin soon."

Which statement does the nurse use to describe to the patient's partner why opioid analgesics are being administered? A. "Opioid analgesics prevent nausea." B. "Opioid analgesics remove the pain of labor." C. "Opioid analgesics help the laboring woman relax between contractions." D. "Opioid analgesics reduce respiratory depression for the laboring woman."

C. "Opioid analgesics help the laboring woman relax between contractions."

A woman is reluctant to perform vulvar self-examinations and asks if they are necessary because she has no family history of reproductive cancers and is not sexually active. What patient education is appropriate for this woman? A. "If you are not sexually active, monthly examinations are not necessary." B. "If your Papanicolaou (Pap) test is normal, you only need to perform self-examination once every 3 months." C. "Performing self-examination now will help you recognize any abnormal changes that may occur in the future." D. "Because you do not have any family history of reproductive cancers, monthly examinations are not necessary."

C. "Performing self-examination now will help you recognize any abnormal changes that may occur in the future."

Which advice would the nurse provide a woman regarding the "baby blues" and postpartum depression as part of discharge teaching? A. "Stay home and avoid outside activities to ensure adequate rest." B. "Be certain that you are the only caregiver for your baby to facilitate infant attachment." C. "Talk about your feelings and seek help if needed." D. "Keep feelings of sadness and adjustment to your new role to yourself."

C. "Talk about your feelings and seek help if needed."

Which statement by a postpartum woman would require the most immediate attention by the nurse? A. "I am so tired I do not know how I am going to make it through the day." B. "I am not hungry. I am going to skip lunch today." C. "The lactation consultant tried to steal my baby." D. "I am never going to feel like myself again."

C. "The lactation consultant tried to steal my baby."

The nurse knows that patient education has been effective when the patient makes which statement about the difference between a tocodynamometer and an intrauterine pressure catheter (IUPC)? A. "Only the tocodynamometer shows my uterine activity." B. "The tocodynamometer is much more accurate than the IUPC." C. "The tocodynamometer is positioned outside my body, while the IUPC is positioned inside my body." D. "The tocodynamometer will be connected to my bedside monitor, but the IUPC will not."

C. "The tocodynamometer is positioned outside my body, while the IUPC is positioned inside my body."

A patient is concerned about the baseline variability in the heart rate of her fetus. Which responses by the nurse describe the significance of baseline variability to the patient? (MA) A. "Variability is an artifact." B. "Variability is a periodic pattern." C. "Variability demonstrates that there is adequate oxygenation of the fetus." D. "Variability suggests that the fetus is able to adapt to the labor process." E. "Variability indicates that the fetus has no congenital abnormalities."

C. "Variability demonstrates that there is adequate oxygenation of the fetus." D. "Variability suggests that the fetus is able to adapt to the labor process."

Which education would the nurse provide a mother who reports latch difficulty with breastfeeding? A. "The newborn is likely sleepy. You should wait until the baby is fully awake." B. "Use a breast pump for a few minutes before feedings to increase nipple size." C. "Wait until the mouth is open wide to insert the nipple into the newborn's mouth." D. "Apply ointment to the nipple. It has a sweet taste and will prompt the newborn to breastfeed."

C. "Wait until the mouth is open wide to insert the nipple into the newborn's mouth."

Which response would the nurse provide the patient who asks why oxygen is being given after the nurse identifies a nonreassuring fetal heart rate? A. "I will call the health care provider to discuss the new care plan." B. "We need to increase the perfusion of the baby's placenta." C. "We need to increase your oxygen, which will increase the baby's oxygen." D. "Don't worry. This happens all of the time, and everything is fine."

C. "We need to increase your oxygen, which will increase the baby's oxygen."

How many ounces of alcohol are considered safe in the first trimester of pregnancy? A. 18 to 22 B. 12 to 16 C. 0 D. 4 to 8

C. 0

The nurse administered magnesium sulfate to a woman experiencing preterm labor, following a health care provider prescription. Which assessment would concern the nurse? A. Urine output of 50 mL B. Oxygen saturation of 95% C. 10 respirations per minute D. Bronchial sounds heard over the body of the sternum

C. 10 respirations per minute

Which situation puts a family at risk for poor adaptation after birth? A. A mother who labored for 8 hours B. A mother who had a planned, repeat cesarean delivery C. A mother who delivered at 32 weeks gestation D. A mother who had a spontaneous vaginal birth of an infant with 9/9 Apgar score

C. A mother who delivered at 32 weeks gestation

The nurse is caring for a patient in the second stage of labor. Which patient condition is most likely to result in the need for an episiotomy? A. A patient with a history of perineal laceration B. A patient receiving oxytocin for induction of labor C. A patient whose fetus is experiencing shoulder dystocia D. A patient who had an episiotomy during a previous delivery

C. A patient whose fetus is experiencing shoulder dystocia

When counseling a patient on the warning signs of complications associated with contraception, the nurse recognizes which patient cue as a warning sign for both oral contraceptive pills (estrogen-containing contraception) and intrauterine devices? A. Fever or chills B. Headaches C. Abdominal pain D. Abnormal vaginal discharge

C. Abdominal pain

A postpartum mother informs the nurse that she is disappointed in the gender of her baby. Which nursing intervention would be most appropriate to support adaptation after birth? A. Instruct the patient not to disclose this to others. B. Remove the newborn from the mother's room and notify the health care provider. C. Acknowledge the patient's feelings and provide emotional support. D. Inform the patient that she should be grateful for a healthy newborn baby.

C. Acknowledge the patient's feelings and provide emotional support.

A woman tests positive for group B streptococcus (GBS) at 36 weeks gestation before a vaginal birth. Which interventions would the nurse include in the plan of care? (MA) A. Retest for GBS upon admission to labor and delivery. B. Instruct the patient to use condoms until the infection clears. C. Administer antibiotics during labor until delivery as prescribed. D. Monitor the neonate for signs and symptoms of respiratory distress. E. Inspect the perineum carefully for lesions upon admission for labor and delivery.

C. Administer antibiotics during labor until delivery as prescribed. D. Monitor the neonate for signs and symptoms of respiratory distress.

Which order during labor augmentation would cause the nurse to question the health care provider? A. Administer oxytocin in lactated Ringer solution per protocol. B. Administer oxytocin in normal saline per protocol. C. Administer oxytocin in dextrose 10% per protocol. D. Administer oxytocin intramuscular per protocol.

C. Administer oxytocin in dextrose 10% per protocol.

A nurse is caring for a hospitalized patient who is receiving intravenous heparin for deep vein thrombosis (DVT). The patient begins to vomit blood. After the heparin has been stopped, which action would the nurse take next? A. Administer vitamin K1. B. Measure the emesis. C. Administer protamine sulfate. D. Perform the Homans sign.

C. Administer protamine sulfate.

Which considerations would the nurse recognize for a pregnant patient with human immunodeficiency virus (HIV)? (MA) A. A second test toward the end of the third trimester is not recommended. B. A vaginal delivery is always contraindicated C. Antiretroviral therapy for the mother and neonate helps reduce transmission at birth. D. Testing all pregnant women for HIV at the beginning of pregnancy is a best practice. E. Breastfeeding should be avoided. F. An elective cesarean birth at 38 weeks helps prevent transmission to the neonate.

C. Antiretroviral therapy for the mother and neonate helps reduce transmission at birth. D. Testing all pregnant women for HIV at the beginning of pregnancy is a best practice. E. Breastfeeding should be avoided. F. An elective cesarean birth at 38 weeks helps prevent transmission to the neonate.

A patient reports a fishy, greenish, malodorous vaginal discharge after intercourse. The nurse understands these findings to be classic symptoms of which condition? A. Genital warts (condyloma) B. Chlamydia C. Bacterial vaginosis (Gardnerella) D. Vaginal yeast infections (Candidiasis)

C. Bacterial vaginosis (Gardnerella)

The nurse understands that which concurrent condition during pregnancy is the leading cause of maternal mortality in the United States today? A. Cytomegalovirus (CMV) B. HIV C. Cardiovascular disease D. Anemia secondary to blood loss

C. Cardiovascular disease

Which clinical conditions must be met before placing a fetal scalp monitor? (MA) A. Patient should not be in labor B. Absence of fetal scalp hair C. Cervix should be at least 2 cm dilated D. The patient's membranes must be ruptured E. High fetal presentation

C. Cervix should be at least 2 cm dilated D. The patient's membranes must be ruptured

The nurse is checking the pad of a patient recovering from a cesarean delivery who has called out, "I just had a large gush of blood!" The nurse does not note an abnormal amount of blood under the patient's legs. Which action would the nurse take next? A. Request that the health care provider evaluate the patient for possible postpartum hemorrhage. B. Administer the as-needed (PRN) dose of methylergonovine. C. Check for pooling of blood under the patient's buttocks and back. D. Explain that this can be normal and evaluate with the next scheduled fundal check.

C. Check for pooling of blood under the patient's buttocks and back.

A woman visiting the health care provider for a routine well-woman visit has a history of stroke and has right-sided hemiparesis, or weakness. The nurse anticipates that the results of which screening procedure may be affected by these symptoms? A. Papanicolaou (Pap) test B. mammography C. Clinical breast examination (CBE) D. Thyroid-stimulating hormone (TSH)

C. Clinical breast examination (CBE)

Which signs would the nurse recognize as indicative of missed abortion? (MA) A. Vaginal bleeding B. Products of conception partially expelled C. Decrease in uterine size D. Absent fetal heart rate E. Subsiding nausea F. Absence of breast tenderness

C. Decrease in uterine size D. Absent fetal heart rate E. Subsiding nausea F. Absence of breast tenderness

Which condition is related to the increased risk for respiratory distress in neonates born to mothers with gestational diabetes? A. Hyperinsulinemia after birth B. Recurrent fetal hypoxia and subsequent increased production of erythrocytes C. Decreased fetal cortisol levels and therefore insufficient surfactant D. Decreased maternal magnesium levels

C. Decreased fetal cortisol levels and therefore insufficient surfactant

Which deceleration is considered a normal finding? A. Variable B. Prolonged C. Early D. Late

C. Early

Which action taken by the patient will reduce discomfort during Leopold maneuvers and make fetal presenting parts easier to feel? A. Standing B. Lying prone C. Emptying the bladder D. Pushing or bearing down with contractions

C. Emptying the bladder

The nurse is developing a standard care plan for a mother after a cesarean delivery. Which step would the nurse plan to implement? A. Maintain patient in a left-lateral recumbent position for the first 24 hours after the cesarean section. B. Teach the patient how to do sitz baths for pain on the second postoperative day. C. Encourage early, frequent ambulation after the surgery. D. Assess arterial blood gases during the first postoperative day.

C. Encourage early, frequent ambulation after the surgery.

A woman is experiencing strong contractions every 1.5 to 2 minutes, feels rectal pressure, and has a large amount of bloody show. At which frequency would the nurse anticipate assessing the fetal heart rate and pattern? A. Every 30 to 45 minutes B. Every 5 to 10 minutes C. Every 15 to 30 minutes D. Every 45 to 60 minutes

C. Every 15 to 30 minutes

Which statement explains how a 5-year-old girl could present with pubic hair growth? A. Estrogen is released prematurely by the anterior pituitary gland. B. Luteinizing hormone (LH) is released in large quantities from the ovaries. C. Gonadotropin-releasing hormone (GnRH) is released prematurely by the hypothalamus. D. Follicle-stimulating hormone (FSH) is released in large quantities from the hypothalamus.

C. Gonadotropin-releasing hormone (GnRH) is released prematurely by the hypothalamus.

A laboring woman wishes to labor in the shower to alleviate pain. Which nonpharmacologic pain relief method is she applying? A. Massage B. Acupressure C. Hydrotherapy D. Relaxation

C. Hydrotherapy

The nurse notes fetal tachycardia and suspects that the patient may be dehydrated. Which nursing action is appropriate to address this nonreassuring finding? A. Consult with the dietician. B. Administer parenteral feeding. C. Increase the rate of intravenous (IV) saline administration. D. Provide the patient an oral (PO) electrolyte replacement.

C. Increase the rate of intravenous (IV) saline administration.

A patient arrives at a birthing center in active labor. Her membranes are still intact, and the health care provider prepares to perform an artificial rupture of membranes (AROM). What will the nurse relay to the patient as the most likely outcome of the procedure? A. Less pressure on the cervix B. Decreased number of contractions C. Increased pressure on the cervix D. The need for more cervical exams

C. Increased pressure on the cervix

Which uterine activity indicators does the intrauterine pressure catheter (IUPC) measure in mm HG? (MA) A. Frequency B. Duration C. Intensity D. Resting tone E. Variability

C. Intensity D. Resting tone

Using the Naegele rule, what is the estimated due date (EDD) for a woman who reports the first day of her last menstrual period (LMP) as September 20? A. May 30 B. June 20 C. June 27 D. July 3

C. June 27

Through which mechanism does nonshivering thermogenesis generate heat in a newborn? A. Vasodilation B. Diminished metabolic activity C. Metabolism of brown fat D. Increased respiratory rate

C. Metabolism of brown fat

For which condition would the nurse teach the family that breastfeeding is contraindicated but feeding expressed breast milk is safe? A. Infant galactosemia B. Mother with human immunodeficiency virus (HIV) C. Mother with active tuberculosis D. Mother with active herpes simplex virus (HSV) lesions on the breast

C. Mother with active tuberculosis

The labor nurse is evaluating the patient's most recent 10-minute segment on the monitor strip and notes a late deceleration. Which explanations most likely explains this occurrence? (MA) A. Spontaneous fetal movement B. Compression of the fetal head C. Placental abruption D. Nuchal cord around the baby's neck E. Vena cava syndrome

C. Placental abruption E. Vena cava syndrome

Which patient data would the nurse validate as part of routine preparation for cesarean delivery to determine adequate fetal maturity? (MA) A. Rh-positive antibody test performed 4 weeks before surgery B. Negative rubella titer performed 6 weeks before surgery C. Positive serum pregnancy test performed at least 36 weeks before surgery D. High serum alpha-fetoprotein (AFP) level performed 26 weeks before surgery E. Ultrasound examination between 6 and 11 weeks of pregnancy

C. Positive serum pregnancy test performed at least 36 weeks before surgery E. Ultrasound examination between 6 and 11 weeks of pregnancy

Which complications of pregnancy are associated with substance misuse? A. Vaginal bleeding B. Gestational diabetes C. Preterm labor D. Intrauterine growth restriction (IUGR) E. Umbilical cord prolapse F. Congenital anomalies

C. Preterm labor D. Intrauterine growth restriction (IUGR) F. Congenital anomalies

During which phase of the endometrial cycle does ovulation occur? A. Menstrual phase B. Proliferative phase C. Secretory phase D. Ischemic phase

C. Secretory phase

A nurse is concerned that her patient is at risk for sepsis. Which findings in the patient's assessment rule out the diagnosis of sepsis? (MA) A. Pulse = 110 beats/min B. 20-mL sample of dark-yellow urine C. T = 97.3°F (36.3°C) D. BP = 100/65 E. RR = 18 and lungs clear bilaterally

C. T = 97.3°F (36.3°C) D. BP = 100/65 E. RR = 18 and lungs clear bilaterally

Which action would the nurse take for a breastfeeding mother who is afebrile but reports erythema and tenderness over her right breast? A. Apply cold compress. B. Administer local anesthetic. C. Teach breast massage technique. D. Administer intramuscular antibiotic.

C. Teach breast massage technique.

Which statement describes how the endometrium responds to the female reproductive cycle? A. The myometrium causes blood loss during menarche. B. The functional layer of the endometrium releases estrogen and progesterone. C. The functional layer of the endometrium is shed during the menstrual period. D. The basal layer of the endometrium regenerates the myometrium each month.

C. The functional layer of the endometrium is shed during the menstrual period.

Which statements are true regarding rubella in pregnancy? (MA) A. Women develop a "slapped-cheek rash." B. Titers less than 1:8 demonstrate immunity. C. The virus can cross the placental barrier. D. Most women of childbearing age in the United States have received this vaccine already. E. If a woman is not immune, she can be offered the vaccine during pregnancy.

C. The virus can cross the placental barrier. D. Most women of childbearing age in the United States have received this vaccine already.

The nurse receives reports on two women in early labor, a nulliparous woman and a multiparous woman. Both are 3 cm dilated. Which statement is true regarding who will enter active labor first? A. The multiparous woman will enter active labor first. B. The nulliparous woman will enter active labor first. C. They will both progress at similar rates. D. The patient whose contractions palpate stronger will enter active labor first.

C. They will both progress at similar rates.

For which reason would a nurse administer a narcotic to a woman at the beginning of a contraction? A. To allow the medication to be transferred to both the laboring woman and the fetus B. To allow for no medication to be transferred to the fetus C. To allow for less medication to be transferred to the fetus D. To decrease the likelihood of maternal hypotension

C. To allow for less medication to be transferred to the fetus

A patient in early pregnancy presents with heavy bleeding and blood clots. Her hemoglobin and hematocrit are low. Based upon the information given, which order would the nurse anticipate? A. Prepare for emergency cesarean birth. B. Administer 650 mg acetaminophen for pain stat. C. Transfuse 1 unit packed red blood cells stat. D. Massage the fundus.

C. Transfuse 1 unit packed red blood cells stat.

Which assessment finding is most concerning for a fertility specialist who is working with a 38-year-old couple who has practiced unprotected intercourse for the past 5 years? A. The man occasionally withdraws his penis during ejaculation B. Spontaneous abortion 3 years ago at 8 weeks' gestation C. Unprotected intercourse for 5 years D. Vegetarian diet

C. Unprotected intercourse for 5 years

The nurse caring for the woman in labor recognizes that maternal hypotension puts the laboring woman at risk for which result? A. Early decelerations B. Fetal dysrhythmias C. Uteroplacental insufficiency D. Spontaneous rupture of membranes

C. Uteroplacental insufficiency

On assessment, the postpartum nurse notes a firm fundus, bright red blood oozing from the vagina, and a saturated perineal pad. What diagnosis would the nurse expect based on these assessment findings? A. Vaginal hematoma B. Placental accerta C. Vaginal laceration D. Uterine inversion

C. Vaginal laceration

On assessment, the nurse learns that a male toddler born with a congenital heart defect lives with his parents and two young siblings in the Midwest. The mother is pregnant with a fourth child. Which additional information would the nurse need to know to evaluate the risk for the fourth child being born with a heart defect? (MA) A. Socioeconomic status of the family B. The mother's family history of tobacco use C. Variations of symptoms because of time or season D. Sex of family members born with a heart defect E. Number of close relatives born with a heart defect

C. Variations of symptoms because of time or season D. Sex of family members born with a heart defect E. Number of close relatives born with a heart defect

Which methods of screening for substance abuse in a pregnant woman are considered acceptable? (MA) A. Verbally screening only at the first prenatal visit B. Collecting a urine sample and sending without consent C. Verbally using a validated screening tool D. Asking the pregnant woman's significant other E. Obtaining patient consent and collecting laboratory samples

C. Verbally using a validated screening tool E. Obtaining patient consent and collecting laboratory samples

When teaching about infant feeding cues in the first days of life, which solution can the nurse consider to encourage the best time to initiate breastfeeding? A. Whenever the newborn begins to cry B. Every 2 hours, around the clock C. When the infant is in the quiet alert state D. When the newborn naturally wakes itself up, regardless of the time interval

C. When the infant is in the quiet alert state

Which signs and symptoms are commonly associated with endometritis? (MA) A. Flank pain B. Breast tenderness C. pelvic pain D. Excessive lochia E. Low-grade fever for 24 hours

C. pelvic pain D. Excessive lochia

What is the method of heat loss with the nursing action that would prevent it? Place a blanket between the newborn and the examination table.

Conduction

Which statement by a pregnant woman would suggest that she needs further teaching about the purpose of childbirth education classes? A. "Childbirth preparation programs will increase my sense of control." B. "Childbirth preparation programs will help prepare my support person to help during labor." C. "Childbirth preparation programs guarantee a pain-free childbirth." D. "Childbirth preparation programs teach distraction techniques."

D. "Childbirth preparation programs teach distraction techniques."

To assess whether there is any improvement in a patient's dysuria, the nurse would ask which question? A. "Do you have to urinate at night?" B. "Do you have blood in your urine?" C. "Do you have to urinate frequently?" D. "Do you have pain when you urinate?"

D. "Do you have pain when you urinate?"

The nurse caring for a patient with a history of a previous cesarean delivery notes a small incision scar on the patient's bikini line (transverse). The patient asks the nurse if a trial of labor will be possible. Which response would the nurse provide? A. "Transverse uterine incisions allow for vaginal births, but the skin and uterine incisions do not always match, so we will have to verify." B. "You have a transverse skin incision, so that means you also have a transverse uterine incision and a trial of labor is possible." C. "Because you have a transverse skin incision, there is a greater chance of having a uterine incision rupture with labor." D. "It is never safe to have a vaginal birth after a cesarean delivery."

A. "Transverse uterine incisions allow for vaginal births, but the skin and uterine incisions do not always match, so we will have to verify."

Which nursing questions are appropriate for a patient 9 days postpartum who feels tired and still has vaginal discharge? (MA) A. "What color is your lochia?" B. "Is your lochia red?" C. "Is there an odor to your lochia?" D. "When did your lochia turn clear?" E. "How often are you changing your peripads?"

A. "What color is your lochia?" C. "Is there an odor to your lochia?" E. "How often are you changing your peripads?"

When teaching a patient with a body mass index (BMI) of 20 about recently prescribed subcutaneous insulin, which statement is correct? A. "You will need to insert the needle at a 45-degree angle to ensure it is delivered to the subcutaneous tissue." B. "If you need to deliver more than 0.5 mL of insulin, you will need to use two injection sites." C. "If you use the no-pinch method, you should use an 8-mm needle." D. "Gently massage the injection site with sterile gauze after administration to ensure the medication is absorbed."

A. "You will need to insert the needle at a 45-degree angle to ensure it is delivered to the subcutaneous tissue."

Which statement by the health care provider is an example of coercive behavior? A. "Your child will be taken away if you continue to use marijuana in your pregnancy." B. "Your child may be born with multiple birth defects and learning disorders if you continue to use alcohol while pregnant." C. "The staff at the hospital will discuss all possible options for pain management in labor if you have too much pain." D. "Using alcohol and cocaine while pregnant is not a healthy behavior for you or your baby."

A. "Your child will be taken away if you continue to use marijuana in your pregnancy."

Which statements regarding the assessment of newborn vital signs are accurate? (MA) A. A heart rate of 146 beats/min is normal. B. A respiratory rate of 66 breaths/min is normal. C. A temperature of 35.8°C (96.4°F) is normal. D. A blood pressure of 70/48 is normal. E. Respiratory rate and heart rate are assessed for a full minute. F. Blood pressure is routinely assessed.

A. A heart rate of 146 beats/min is normal. D. A blood pressure of 70/48 is normal. E. Respiratory rate and heart rate are assessed for a full minute.

Which patient would benefit from prenatal genetic counseling? A. A patient with sickle cell disease (SCD) B. A patient who is having difficulty conceiving C. A patient with history of two multifetal pregnancies D. A patient with a history of miscarriage resulting from polyploidy

A. A patient with sickle cell disease (SCD)

Which postpartum women may require nursing care for ineffective bladder elimination? (MA) A. A woman with third-degree perineal laceration and significant edema B. A woman who received epidural anesthesia during labor C. A woman with a first-degree perineal laceration and slight edema D. A woman with an episiotomy and a firm, midline fundus at the umbilicus E. A woman who delivered by cesarean birth and had an indwelling catheter removed

A. A woman with third-degree perineal laceration and significant edema B. A woman who received epidural anesthesia during labor E. A woman who delivered by cesarean birth and had an indwelling catheter removed

Which intervention would the nurse implement for a 10-minute-old healthy newborn? A. Administer vitamin K. B. Assess blood pressure. C. Obtain serum glucose level. D. Perform heel stick for universal newborn screening.

A. Administer vitamin K.

A G5/P4 is experiencing painful contractions that start in her back every 10 minutes for the last hour. How would the telephone triage nurse advise this woman? A. Advise the patient to report to the hospital for labor and delivery. B. Tell the patient to call an ambulance. C. Instruct the woman to come to the hospital when the contractions are every 5 minutes. D. Inform the patient that she is experiencing Braxton Hicks contractions.

A. Advise the patient to report to the hospital for labor and delivery.

A patient with a family history of developmental disability refuses a recommended amniocentesis without explanation. Which types of patient education are appropriate? (MA) A. Affirm that the patient has the right to refuse any procedure. B. Explain to the patient that amniocentesis can be used to diagnose developmental disabilities. C. Inform the patient that refusing the amniocentesis may hinder the effectiveness of prenatal care. D. Inform the patient that prenatal diagnosis can give the family more time to prepare for a child with special needs. E. Remind the patient that the health care provider ordering the procedure may simply order a test besides amniocentesis if the patient refuses.

A. Affirm that the patient has the right to refuse any procedure. B. Explain to the patient that amniocentesis can be used to diagnose developmental disabilities. D. Inform the patient that prenatal diagnosis can give the family more time to prepare for a child with special needs.

Which assessments would the nurse include during the initial assessment of a newborn after birth? (MA) A. Airway patency B. Heart sounds C. Head circumference D. Length E. Color

A. Airway patency B. Heart sounds E. Color

Which conditions are significant causes of deep vein thrombosis (DVT)? A. Altered blood coagulation B. Stasis of blood C. High-grade fever D. Positive Homans sign E. Vessel wall injury

A. Altered blood coagulation B. Stasis of blood E. Vessel wall injury

Which measures should the nurse ensure are available and ready before a multiple gestation twin delivery? (MA) A. An operating room set up for vaginal and cesarean delivery B. Neonatal health care providers capable of advanced resuscitation for each baby C. A single infant warmer with supplies for each infant D. Two separate infant warmers with separate supplies for each baby E. A fetal monitor with the capacity to monitor two babies at the same time

A. An operating room set up for vaginal and cesarean delivery B. Neonatal health care providers capable of advanced resuscitation for each baby D. Two separate infant warmers with separate supplies for each baby E. A fetal monitor with the capacity to monitor two babies at the same time

A woman reports dissatisfaction when using her vaginal hormonal ring and asks about starting another method of contraception. Which steps would the nurse take? (MA) A. Ask the woman what qualities she is looking for in a contraceptive method. B. Provide the woman with information about multiple other contraceptive methods. C. Ask the woman what she finds dissatisfying about her current contraceptive method. D. Instruct the woman about proper vaginal ring usage and plan on discussing switching methods at the next visit. E. Inform the woman that learning how to use another contraceptive method will distract her from learning how to use her current method correctly.

A. Ask the woman what qualities she is looking for in a contraceptive method. B. Provide the woman with information about multiple other contraceptive methods. C. Ask the woman what she finds dissatisfying about her current contraceptive method.

A woman delivered a 9-lb, 10-oz baby 1 hour ago. When the nurse arrives to perform the 15-minute assessment, the patient says that she "feels all wet underneath." The nurse discovers that both perineal pads are completely saturated and that the patient is lying in a 6-inch-diameter puddle of blood. After calling for help, which action would the nurse take next? A. Assess the fundus for firmness. B. Estimate the blood loss by weighing the perineal pads. C. Check the perineum for lacerations D. Manually remove any contents in the uterus.

A. Assess the fundus for firmness.

The nurse caring for a patient in the postanesthesia care unit (PACU) notes that a patient is having bradypnea after general anesthesia. Which immediate interventions are indicated to determine the next actions by the nurse? (MA) A. Assess the patient's level of consciousness. B. Ensure the pulse oximeter is on and within normal limits. C. Call the anesthesiologist for possible intubation. D. Administer the prescribed dose of naloxone. E. Continue to monitor the patient, as bradypnea is a normal response to general anesthesia.

A. Assess the patient's level of consciousness. B. Ensure the pulse oximeter is on and within normal limits.

Which experiences influence how a new father identifies with his new role? (MA) A. Attitude about fatherhood B. Relationship with siblings C. Relationship with own father D. Childhood experiences E. Job security

A. Attitude about fatherhood C. Relationship with own father D. Childhood experiences

A woman has a 6-month history of abnormal Papanicolaou (Pap) tests. Which screening procedures should be used to further examine any persistent abnormalities? (MA) A. Biopsy B. Colpscopy C. Pelvic examination D. Bimanual examination E. Vulvar self-examination

A. Biopsy B. Colpscopy

The nurse is completing a medication reconciliation for a pregnant patient with chronic hypertension. Which medication would cause the nurse to alert the health care provider? A. Captopril (angiotensin-converting enzyme [ACE] inhibitor) 25 mg bid B. Labetalol (beta blocker) 200 mg tid C. Nifedipine (calcium channel blocker) 30 mg daily D. Oral (PO) hydralazine (vasodilator) 25 mg tid

A. Captopril (angiotensin-converting enzyme [ACE] inhibitor) 25 mg bid

Which newborn assessment finding is considered a normal variant? A. Caput succedaneum B. Peripheral acrocyanosis C. Minimal tearing D. Nostrils patent bilaterally

A. Caput succedaneum

Which characteristics are associated with sickle cell anemia? (MA) A. Causes frequent hospitalization throughout the life span B. Characterized by distorted and decreased erythrocytes C. Can lead to preterm birth D. causes severe pain E. Commonly leads to fetal neural tube defects F. May be cured with diet and supplementation

A. Causes frequent hospitalization throughout the life span B. Characterized by distorted and decreased erythrocytes C. Can lead to preterm birth D. causes severe pain

While evaluating an external monitor tracing of a woman who just received regional analgesia for pain management, the nurse notes recurrent late decelerations. Which action should the nurse take next? A. Change the woman's position. B. Notify the obsteric provider C. Assist in amnioinfusion D. Rupture the membranes and insert a scalp electrode.

A. Change the woman's position.

Which risk factor places a woman at risk for subinvolution of the uterus? A. Chorioamnionitis B. Forceful traction of the umbilical cord C. Vacuum extraction D. Precipitous delivery

A. Chorioamnionitis

Which medication is indicated for ovulation induction? A. Clomiphene citrate B. Follitropins C. Exogenous progesterone D. Metoclopramide

A. Clomiphene citrate

The antepartum nurse is caring for a patient with a family history of neural tube defects. The patient declines genetic counseling after listening to the health care provider's explanation. Which nursing intervention is appropriate for this patient? A. Continue to the next part of the visit. B. Inform the patient that declining genetic counseling may lead to inadequate preparation if the infant is born with a defect. C. Provide the patient with printed material regarding genetic counseling to take home should the patient have a change of mind. D. Explain to the patient that the test must be done because a family history of neural tube defects indicates a need for genetic counseling.

A. Continue to the next part of the visit.

Which fetal condition can cause a fetal heart rate (FHR) of <60 beats/min for more than 60 seconds that does not quickly return to baseline? A. Cord compression B. Fetal head compression C. Uteroplacental insufficiency D. No compromise; variable decelerations are normal

A. Cord compression

Which factors could improve a woman's experience with pain during labor? (MA) A. Cultural factors B. Financial factors C. Environmental factors D. Psychological factors E. Physiologic factors

A. Cultural factors C. Environmental factors D. Psychological factors E. Physiologic factors

When teaching a childbirth class for first-time parents, the nurse includes which benefits of breathing techniques used during labor? (MA) A. Decreases sensation of pain B. Requires increased energy with continued use C. Provides a different focus during contractions D. Can also substitute for pharmacologic therapies E. Can be used with other nonpharmacologic techniques

A. Decreases sensation of pain C. Provides a different focus during contractions D. Can also substitute for pharmacologic therapies E. Can be used with other nonpharmacologic techniques

Which instructions would the nurse include when educating the family of a patient admitted for preeclampsia? (MA) A. Dim the lights. B. Limit activity C. Increase fluid intake. D. Turn off the television. E. Limit the number of visitors F. Decrease protein intake.

A. Dim the lights. B. Limit activity D. Turn off the television. E. Limit the number of visitors

A G1/P0 patient arrives for elective induction of labor at 39 weeks and is 1 to 2 cm dilated and 50% effaced. The fetal station is −3 with a posterior and firm cervix. Which order is most appropriate based on this patient's Bishop score? A. Discharge to home with follow-up in 1 week at the office. B. Administer oxytocin and titrate per protocol. C. Assist with amniotomy and initiate oxytocin as prescribed. D. Initiate electronic fetal monitoring and insert peripheral intravenous device.

A. Discharge to home with follow-up in 1 week at the office.

Which nursing actions are appropriate when the fetal monitor shows a pattern of late decelerations? (MA) A. Discontinue oxytocin. B. Provide water to the patient. C. Administer oxygen by face mask. D. Reposition the patient onto her side. E. Continue monitoring and document findings.

A. Discontinue oxytocin. C. Administer oxygen by face mask. D. Reposition the patient onto her side.

While monitoring a patient receiving oxytocin for augmentation of labor, the nurse notes tachysystole with recurrent late decelerations and minimal variability on the electronic fetal monitor. Which actions are appropriate? (MA) A. Discontinue the oxytocin infusion. B. Reposition the patient on her side. C. Administer an intravenous bolus of fluid per protocol. D. Administer 100% oxygen via tight face mask. E. Notify the health care provider. F. Place the patient in semi-Fowler position and continue to monitor.

A. Discontinue the oxytocin infusion. B. Reposition the patient on her side. C. Administer an intravenous bolus of fluid per protocol. D. Administer 100% oxygen via tight face mask. E. Notify the health care provider.

Which effects can a full bladder have on the uterus in the postpartum period? (MA) A. Displaces the uterus B. Increases uterine tone C. Promotes a boggy uterus D. Inhibits uterine involution E. Increases uterine involution

A. Displaces the uterus C. Promotes a boggy uterus D. Inhibits uterine involution

Which intervention can prevent nausea and vomiting in pregnancy? A. Eat small, frequent meals throughout the day. B. Eat one meal per day until the morning sickness subsides. C. Fast food is okay throughout the pregnancy as long as dairy is avoided. D. Drink plenty of fluids, especially carbonated beverages, until the morning sickness subsides.

A. Eat small, frequent meals throughout the day.

The nurse admitting a patient for preeclampsia notes that the patient's platelet count is 95,000. Which additional laboratory result would support the diagnosis of HELLP syndrome? A. Elevated AST/ALT B. Low hemoglobin C. Increased glomerular filtration rate (GFR) D. Decreased serum lactate (LD)

A. Elevated AST/ALT

The nurse is assessing a 3-day-postpartum breastfeeding woman. Her breasts are firm and warm to the touch. When asked when she last fed the infant, her reply is, "I fed the baby last evening, but I let the nurses feed him in the nursery last night because I needed to rest." Which action would the nurse take to prevent the woman from developing mastitis? A. Encourage the woman to breastfeed her infant more frequently. B. Have the woman massage her breasts hourly. C. Obtain a prescription to culture her expressed breast milk. D. Take the temperature and pulse rate of the woman.

A. Encourage the woman to breastfeed her infant more frequently.

Which statement is consistent with the diagnosis of endometriosis? A. Endometriosis may cause infertility. B. Endometriosis is associated with a higher incidence of cervical cancer. C. Endometriosis can be diagnosed with a blood test. D. Endometriosis is concentrated in the uterus.

A. Endometriosis may cause infertility.

The restoration of the endometrium is dependent upon which factor? A. Estrogen stimulation from the ovarian follicles B. Estrogen stimulation from the corpus luteum C. Estrogen stimulation from luteinizing hormone (LH) D. Estrogen stimulation from the hypothalamus

A. Estrogen stimulation from the ovarian follicles

Which factors contribute to the severity of withdrawal symptoms in neonatal abstinence syndrome? (MA) A. Exposure amount B. Specific opioid exposure C. Exposure duration D. Exposure timing E. Exposure route

A. Exposure amount C. Exposure duration D. Exposure timing

The nurse understands that which physiologic changes in the thyroid are considered abnormal in pregnancy? (MA) A. Fatigue, edema, and muscle aches may be the result of thyroid activity in pregnancy. B. The thyroid gland enlarges and becomes palpable. C. Thyroid-stimulating hormone (TSH) decreases in the first trimester. D. Free thyroxine (T4) levels increase. E. Increased free thyroxine (T4) levels lead to weight loss and goiter.

A. Fatigue, edema, and muscle aches may be the result of thyroid activity in pregnancy. B. The thyroid gland enlarges and becomes palpable. E. Increased free thyroxine (T4) levels lead to weight loss and goiter.

Which health care provider prescription would the nurse anticipate for a newborn whose heelstick reveals a glucose reading of 49 mg/dL? A. Feed the newborn breast milk or formula. B. Offer glucose water. C. Administer intravenous (IV) fluids. D. Reassess blood glucose in 30 minutes.

A. Feed the newborn breast milk or formula.

Which assessments may be performed using ultrasonography? A. Fetal anatomy B. Fetal Rh status C. Gestational age D. chromosomal disorders E. Fetal heart activity F. Amniotic fluid index

A. Fetal anatomy C. Gestational age D. chromosomal disorders E. Fetal heart activity F. Amniotic fluid index

A woman is visibly anxious, refuses position changes in labor, and verbalizes, "I'm afraid I'm going to die." The nurse knows it is important to help the woman relax through effective coaching because of which responses to stress experienced in labor? (MA) A. Fetal intolerance to labor B. Bradycardia related to a vagal response C. Release of catecholamines in the mother's body D. Inhibited uterine contractions because of decreased blood flow to the placenta E. Umbilical cord prolapse

A. Fetal intolerance to labor C. Release of catecholamines in the mother's body D. Inhibited uterine contractions because of decreased blood flow to the placenta

Which supplement would the nurse recommend to prevent fetal neural tube defects as a result of pernicious anemia? A. Folic acid B. calcium carbonate C. Magnesium hydroxide D. red raspberry leaf tea

A. Folic acid

Which reason explains why adequate amounts of folic acid are needed during pregnancy? A. Folic acid prevents birth defects. B. Folic acid prevents preterm labor. C. Folic acid increases iron absorption. D. Folic acid helps maintain the pregnancy.

A. Folic acid prevents birth defects.

Which statements are true regarding hyperglycemia? (MA) A. Glucose levels in the blood may be elevated, but at the cellular level glucose may be low with some forms of diabetes. B. In gestational diabetes, hyperglycemia always leads to polydipsia. C. The body attempts to dilute the glucose load, resulting in polyuria. D. Glycosuria can occur. E. Glucose is stored in the pancreas when hyperglycemia occurs.

A. Glucose levels in the blood may be elevated, but at the cellular level C. The body attempts to dilute the glucose load, resulting in polyuria. D. Glycosuria can occur.

The nurse is responsible for intake history at an obstetrics and gynecology (OB/GYN) office. Which medical conditions would the nurse recognize as high-risk conditions that could have adverse outcomes on a pregnancy? (MA) A. Hemophilia B. Type 1 diabetes mellitus C. HIV D. Chronic hypertension E. Xeroderma

A. Hemophilia B. Type 1 diabetes mellitus C. HIV D. Chronic hypertension

Which viral infection can result in neonatal brain complications? A. Herpes infection B. rubella infection C. Varicella-zoster infection D. Cytomegalovirus infection

A. Herpes infection

Which advantages are associated with human milk (expressed and/or donor milk) feeding? (MA) A. Improved Infant immunity B. Lowest infant feeding-related costs C. Ability to track amount of milk consumed D. Easy storage and shelf life E. Others able to feed infant F. Promotes maternal-infant attachment

A. Improved Infant immunity C. Ability to track amount of milk consumed E. Others able to feed infant

Which physiologic change in a pregnant woman is associated with nasal congestion? A. Increased estrogen levels B. Increased oxytocin levels C. Increased prolactin levels D. Increased progesterone levels

A. Increased estrogen levels

Which care would the nurse anticipate providing an infant with a brachial plexus injury? (MA) A. Intermittent immobilization B. Managing seizures C. Range-of-motion exercises D. Mechanical ventilation E. Gentle manipulation

A. Intermittent immobilization C. Range-of-motion exercises E. Gentle manipulation

The nurse is caring for a patient who has just received genetic counseling and is visibly distressed. The patient tearfully explains to the nurse that the counselor informed her there was a 50% chance of her child having cystic fibrosis and asks, "What should I do about my pregnancy?" Which response from the nurse is appropriate for this patient? A. "Did the counselor say if there was anything you could do to lower the chance?" B. "We can get you information about adoption, if you think you might want to look at it." C. "A 50% chance isn't very high. There's an equal chance your baby will be perfectly healthy." D. "I know you're faced with a difficult decision, but I'm here to support the choices you make."

D. "I know you're faced with a difficult decision, but I'm here to support the choices you make."

Which patient statement would signal a contraindication for a transcervical chorionic villus sampling (CVS) procedure? A. "My health care provider told me I have preeclampsia." B. "My last ultrasound said I'm 10 weeks pregnant." C. "I had a baby before this one, but it had neurological defects." D. "I took fluconazole yesterday for a yeast infection."

D. "I took fluconazole yesterday for a yeast infection."

Which statement by a postpartum woman indicates that the teaching by the nurse regarding thrombus formation was successful? A. "I'll stay in bed for the first 3 days after my baby is born." B. "I'll keep my arms elevated with pillows." C. "I'll sit in my rocking chair most of the time." D. "I'll put my support stockings on every morning before rising."

D. "I'll put my support stockings on every morning before rising."

A woman who is at 38 weeks gestation reports suspected signs of labor to the triage nurse. Which statement by the nurse supports the beginning of true labor? A. "Your contractions will decrease with activity." B. "The contractions will be mild and more annoying than painful." C. "You will feel the contractions in your front pelvic area and not in your back." D. "Labor contractions will occur in a consistent pattern that increases in frequency, duration, and intensity."

D. "Labor contractions will occur in a consistent pattern that increases in frequency, duration, and intensity."

A patient with no family history of genetic abnormalities asks the nurse if there may still be a need for prenatal genetic counseling. Which patient statement indicates that genetic counseling may be necessary? A. "My partner is 14 years older than me." B. "I was adopted by my biological mother's sister." C. "One of my children went deaf in one ear after an ear infection." D. "My family and I practice a religion that does not allow marriage outside of our religion."

D. "My family and I practice a religion that does not allow marriage outside of our religion."

Which statement by a maternal home care nurse reflects the correct approach to addressing potential and actual postpartum depression in maternal patients? A. "We include education about postpartum depression to women if they have a history of depression." B. "If we suspect a woman may have developed postpartum depression, then we provide specialized education about that topic." C. "Because emotional disorders and imbalances are a very sensitive subject, we try not to offend patients by routinely bringing up the topic of postpartum depression." D. "Teaching about postpartum depression is a routine part of education for all pregnant patients."

D. "Teaching about postpartum depression is a routine part of education for all pregnant patients."

Which statements by the nurse are the most appropriate to make to a patient who has recently had a miscarriage? (MA) A. "When it is meant to happen, it will happen." B. "The fetus likely had a congenital anomaly that was incompatible with life." C. "Don't worry; nature has a way of taking care of mistakes. It is for the best." D. "This is often a consequence of unavoidable factors, and it is important to realize this is not your fault." E. "I am very sorry for your loss. Having a miscarriage must be so emotionally painful. How are you feeling?"

D. "This is often a consequence of unavoidable factors, and it is important to realize this is not your fault." E. "I am very sorry for your loss. Having a miscarriage must be so emotionally painful. How are you feeling?"

After a primipara's admission to the labor and delivery suite, the nurse assesses her discharge needs. She will be discharged home 4 days after a cesarean delivery. Which questions would the nurse ask the patient? (MA) A. "Have you ever had anesthesia before?" B. "Do you have any allergies to shellfish?" C. "Do you have a name picked out?" D. "Will you have help when you go home?" E. "Are there many stairs in your home?"

D. "Will you have help when you go home?" E. "Are there many stairs in your home?"

Which education would the nurse provide the mother of a newborn and 2-year-old who expresses concern about the baby's sleep schedule? A. "You should set a strict schedule for your baby's naps." B. "During active sleep, if the baby moves, she is waking up." C. "Feeding your baby during the drowsy state will help promote sleep." D. "Your baby will not be disturbed by other children during quiet sleep."

D. "Your baby will not be disturbed by other children during quiet sleep."

Which finding would the nurse recognize as indicating progression of preeclampsia to eclampsia? A. Proteinuria B. Weight gain in the absence of edema C. Pitting edema in the absence of weight gain D. A generalized seizure not attributed to other causes

D. A generalized seizure not attributed to other causes

Which nursing finding requires intervention when assessing a postpartum woman who delivered by cesarean birth? A. A small amount of lochia B. Foley catheter output of 200 mL/hour C. Abdominal dressing with a small amount of serosanguinous drainage D. Distended abdomen with no bowel sounds auscultated

D. Distended abdomen with no bowel sounds auscultated

A patient reports a feeling of lightness in her abdomen, increasing whitish, thin, and odorless vaginal discharge, and occasional contractions that go away after an hour or so. Which action would the nurse take next? A. Advise the patient to report to the birthing center. B. Suggest routine sexual transmitted infection counseling. C. Call the ambulance because she is experiencing an emergency. D. Educate the patient on the premonitory signs of labor.

D. Educate the patient on the premonitory signs of labor.

Which nursing intervention can help reduce a family's stress related to adaptation after birth? A. Instruct parents about neonatal cues. B. Provide extra attention to siblings that visit. C. Provide opportunities for the parents to perform newborn care. D. Encourage the parents to identify social support to assist them after discharge.

D. Encourage the parents to identify social support to assist them after discharge.

Which fetal heart rate finding may result from maternal fever? A. No expected changes B. Prolonged decelerations C. Fetal heart rate of 90 beats/min for 15 minutes D. Fetal heart rate of 180 beats/min for 12 minutes

D. Fetal heart rate of 180 beats/min for 12 minutes

A patient is experiencing intense back pain and discomfort during active labor. Which position would the nurse encourage the patient to assume? A. Standing straight B. Side-lying C. Semi-sitting D. Hands and knees

D. Hands and knees

Which uterine contraction strength classification is used when labor is measured at >500 Montevideo units (MA)? A. Normal B. Moderate C. Hypotonic D. Hypertonic

D. Hypertonic

Which statement explains how intrathoracic pressure contributes to newborn respiratory adaptation? A. It activates chemoreceptors in the carotid arteries and aorta. B. It activates skin sensors that send impulses to stimulate the respiratory center. C. It causes a drop in prostaglandin levels, thereby reducing inhibition of respirations. D. It forces fetal lung fluid out of the alveoli and into interstitial spaces.

D. It forces fetal lung fluid out of the alveoli and into interstitial spaces.

In which way is the hypothalamus responsible for sexual maturation? A. It produces estrogen to stimulate growth of breast tissue. B. It releases luteinizing hormone (LH) to stimulate growth of the ovaries. C. It produces follicle-stimulating hormone (FSH) to stimulate maturation of the follicles. D. It releases gonadotropin-releasing hormone (GnRH) to stimulate the anterior pituitary gland.

D. It releases gonadotropin-releasing hormone (GnRH) to stimulate the anterior pituitary gland.

Which assessment finding explains why a 25-year-old woman reports anovulation for the past 1.5 years? A. Breast tenderness B. Previous ectopic pregnancy C. Small fibroid tumor D. Marathon training

D. Marathon training

What are the four phases of the endometrial cycle that describe changes to the endometrium in the correct order? A. Proliferative, secretory, ischemic, and menstrual phases B. Proliferative, menstrual, secretory, and ischemic phases C. Menstrual, secretory, proliferative, and ischemic phases D. Menstrual, proliferative, secretory, and ischemic phases

D. Menstrual, proliferative, secretory, and ischemic phases

Which maternal adaptation is appropriate when understanding and guiding a patient through her acceptance of pregnancy? A. Anxiety is an abnormal adaptation. B. Depression is a normal adaptation. C. Narcissism is an abnormal adaptation. D. Mood changes are a normal adaptation.

D. Mood changes are a normal adaptation.

Which nursing instruction is appropriate when recommending food or drink to enhance the absorption of iron during pregnancy? A. Milk B. Water C. Cheese D. Orange juice

D. Orange juice

Which action would the nurse take to improve placental blood flow immediately after administration of an epidural? Place the woman flat on her back B. PLacing the woman prone C. Place the woman in the high-Fowler position D. Place a wedge under the woman's right hip

D. Place a wedge under the woman's right hip

When assessing the neonate born to a mother with preeclampsia, the nurse knows that low birth weight may be attributed to which factor? A. Maternal diabetes B. LImited maternal diet C. Maternal hypotension D. Poor placental perfusion

D. Poor placental perfusion

Which nonreassuring fetal heart rate (FHR) pattern includes a reduction in FHR of ≥15 beats/min for more than 2 minutes? A. Accelerations B. Tachycardia C. Early decelerations D. Prolonged decelerations

D. Prolonged decelerations

A woman in labor is given an opioid analgesic for pain. Which vital sign is a priority for the nurse to monitor in the newborn infant? A. Pulse B. Temperature C. Blood pressure D. Respiratory rate

D. Respiratory rate

Which maternal-infant blood type combination may lead to maternal sensitization during pregnancy? A. Maternal blood type AB; fetal blood type B B. Rh-positive mother; Rh-positive fetus C. Maternal blood type O; fetal blood type A D. Rh-negative mother; Rh-positive fetus

D. Rh-negative mother; Rh-positive fetus

Which food or beverage is the most important for a pregnant woman at 32 weeks of gestation to consume before, during, and after her tennis matches? A. Extra protein sources, such as peanut butter crackers B. Salty foods to replace lost sodium C. Caffeinated energy water D. Several glasses of water

D. Several glasses of water

Which nursing instruction is appropriate to give a pregnant woman who will be travelling in a car for 8 hours? A. Decrease fluid intake. B. Drive with minimal stops. C. Avoid travel by automobile. D. Stop frequently to walk.

D. Stop frequently to walk.

Which maternal or fetal condition might require an amnioinfusion? A. The mother has polyhydramnios (excess amniotic fluid). B. The fetal heart rate (FHR) is 140 beats/min. C. The patient's blood pressure has dropped. D. The fetus has experienced umbilical cord compression.

D. The fetus has experienced umbilical cord compression.

A pregnant woman arrives at the emergency department, and after completing a vaginal examination the nurse midwife states that the patient is 5 cm dilated and 75% effaced and the fetus is at −3 station. Which statement is accurate regarding this assessment? The patient may initiate pushing efforts. B. The cervix is fully dilated. C. The cervix is fully effaced. D. The fetus is not engaged within the maternal pelvis.

D. The fetus is not engaged within the maternal pelvis.

When assessing a patient with preeclampsia on magnesium sulfate for seizure prophylaxis, the nurse notes the patient demonstrates hyperreflexia (+4). How does the nurse interpret this patient cue? A. The nurse should administer a 500-mL bolus of lactated Ringer's solution. B. The nurse should interpret this as normal and continue the therapy as prescribed. C. The nurse should request a prescription for calcium gluconate immediately. D. The nurse should question the health care provider regarding the current dose of magnesium sulfate.

D. The nurse should question the health care provider regarding the current dose of magnesium sulfate.

When administering corticosteroids to a patient in preterm labor, which information is relevant to the nurse regarding the patient's history? A. The patient is 33 weeks' gestation. B. The patient suffers from chronic hypertension. C. The patient's membranes are ruptured. D. The patient has type 1 diabetes.

D. The patient has type 1 diabetes.

Which reasoning is appropriate when documenting reported fetal movement as a presumptive sign of pregnancy? A. The symptoms have no alternative causes. B. The symptoms could have alternative causes such as uterine tumors, polyps, or ascites. C. The symptoms are associated with a "woman's intuition." D. The symptoms could have alternative causes such as peristalsis, flatus, or abdominal muscle contractions.

D. The symptoms could have alternative causes such as peristalsis, flatus, or abdominal muscle contractions.

Which situation demonstrates how an environmental factor could negatively affect a woman's experience of pain during labor? A. The woman experienced tearing of the perineum during delivery. B. The woman had a complicated delivery with her previous child. C. The woman's family views vocalizing about pain as "complaining." D. The woman has no support at her bedside.

D. The woman has no support at her bedside.

Which phrase describes the purpose of the luteal phase of the female reproductive cycle? A. To release the mature ovum B. To promote the growth of the follicle C. To enable menstrual flow D. To prepare the endometrium for the ovum

D. To prepare the endometrium for the ovum

What is the method of heat loss with the nursing action that would prevent it? Keep newborn dry and covered.

Evaporation

the steps in the order in which they would be applied to perform Leopold maneuvers.

Palpate the uterine fundus. Palpate for the fetal back. Palpate the suprapubic area to confirm presentation. Determine if head is flexed (vertex) or extended (face).

List relevant patient cues in the order the nurse recognizes them in a primipara during labor as the patient progresses from the first stage of labor to the second stage of labor.

Patient wakes up and feels mild contractions at home. Cervix is thick and 50% effaced on assessment at the hospital. Contractions become closer and coordinated. Cervix is 6 cm dilated. Patient has bloody show and reports rectal pressure.

What is the method of heat loss with the nursing action that would prevent it? Move the crib away from the window or outside wall.

Radiation

Match the expected event with the stage of labor. Complete cervical dilatation and effacement to birth of the baby

Second stage

Match the expected event with the stage of labor. Birth of the baby to expulsion of the placenta

Third stage

A patient in the early postpartum period asks, "How do I know if my baby is getting enough breast milk? I don't feel anything there." What are the desirable outcomes? (MA) A. "A good way to tell if your baby is taking in enough breast milk is if you continue to see enough wet and soiled diapers." B. "If your baby is gaining weight with each checkup at the pediatrician, it usually means your baby is getting enough breast milk." C. "If your baby actively participates in a feeding, you can hear swallowing and your baby seems content in between feeding. These are all signs that your baby is satisfied with the quantity of breast milk your body is producing." D. "If your baby feeds for at least 10 minutes on each breast with every feed, you know your baby is getting enough breast milk." E. "When your milk comes in, your breasts will feel full and most women will feel a milk letdown. This is a good sign that your body is producing enough breast milk for your baby."

A. "A good way to tell if your baby is taking in enough breast milk is if you continue to see enough wet and soiled diapers." B. "If your baby is gaining weight with each checkup at the pediatrician, it usually means your baby is getting enough breast milk." C. "If your baby actively participates in a feeding, you can hear swallowing and your baby seems content in between feeding. These are all signs that your baby is satisfied with the quantity of breast milk your body is producing." E. "When your milk comes in, your breasts will feel full and most women will feel a milk letdown. This is a good sign that your body is producing enough breast milk for your baby."

Which nursing response is appropriate for a perimenopausal woman who states she no longer needs to use birth control because her periods are irregular? A. "Although your periods are irregular, you can still become pregnant." B. "You are right. You are very unlikely to get pregnant." C. "Because your periods are irregular, birth control is not necessary." D. "Are you worried about becoming pregnant?"

A. "Although your periods are irregular, you can still become pregnant."

The laboring patient becomes verbally abusive when denied a cigarette to relax. Which response by the nurse is most appropriate? A. "I know this is scary and may be causing some anxiety, but I'll help you the best way that I can." B. "I'm sorry, but smoking isn't allowed inside the hospital." C. "I'm sorry, but smoking is bad for you and the baby." D. "Please don't talk to me like that—I'm here to help you."

A. "I know this is scary and may be causing some anxiety, but I'll help you the best way that I can."

A postpartum patient verbalizes that she is feeling overwhelmed about going back to work full-time after 6 weeks of maternity leave. Which statement by the nurse would be supportive of the mother and her concerns? A. "I know you're overwhelmed, and it's common to feel that way. What kind of support do you have from friends and family?" B. "I'm concerned you feel that way. I think you should be evaluated for postpartum depression." C. "You might want to consider cutting back your hours at work." D. "Lots of mothers feel that way. You'll learn to adjust."

A. "I know you're overwhelmed, and it's common to feel that way. What kind of support do you have from friends and family?"

Which statement would a nurse anticipate from a postpartum mother in the letting-go phase? A. "It seems strange not to feel the baby kicking in my uterus." B. "Can you feed the infant a bottle? I need to sleep." C. "Am I putting the diaper on correctly?" D. "I can't believe that the delivery happened so quickly."

A. "It seems strange not to feel the baby kicking in my uterus."

A primigravida Hispanic patient asks why she needs to be screened for gestational diabetes when she is not a diabetic. Which response by the nurse is appropriate? A. "Many patients with gestational diabetes show no symptoms. It is an important way to help you have a healthy pregnancy." B. "We test all patients for diabetes because today's diet makes you more likely to get diabetes." C. "Your race makes you more likely to develop gestational diabetes, so we are screening you just in case." D. "Because this is your first pregnancy, we don't know what to expect, so we are going to test you just in case."

A. "Many patients with gestational diabetes show no symptoms. It is an important way to help you have a healthy pregnancy."

An obese woman reports having no body image-related distress and asks if being obese is "really that bad." Which preventive counseling statements are appropriate? (MA) A. "Obesity puts you at risk for multiple types of cancer." B. "The only disease you are at significant risk for is diabetes." C. "If you're planning to have children, you should be aware that obesity puts you at risk for fertility problems, miscarriage, and stillbirth." D. "Inactivity related to obesity puts you at greater risk for stroke." E. "As long as you are comfortable with your body image, there is no reason for you to alter your diet or exercise habits."

A. "Obesity puts you at risk for multiple types of cancer." C. "If you're planning to have children, you should be aware that obesity puts you at risk for fertility problems, miscarriage, and stillbirth." D. "Inactivity related to obesity puts you at greater risk for stroke."

Which recommendations would the nurse make to the family whose newborn falls asleep after 2 to 3 minutes of breastfeeding? (MA) A. "The baby needs to be awakened, so unwrap the newborn and gently stroke his palms or soles." B. "This is a sign that the newborn is full, so breastfeeding should stop until the baby shows hunger cues." C. "Change the newborn's diaper to wake your newborn up." D. "The newborn is having trouble establishing breastfeeding, so switch to bottle feeding." E. "This is a sign of nipple confusion, so provide pacifiers between feedings."

A. "The baby needs to be awakened, so unwrap the newborn and gently stroke his palms or soles." C. "Change the newborn's diaper to wake your newborn up."

Which education would the nurse provide a patient who is worried that the intrauterine catheter will hurt the baby? A. "The catheter is placed in the uterus next to the baby." B. "Don't be nervous; everything will be fine!" C. "The catheter attaches only to the outer layer of skin on the baby's head." D. "This is the equipment needed for high-risk pregnancies."

A. "The catheter is placed in the uterus next to the baby."


Conjuntos de estudio relacionados

Chapter 11.2: DECISION-MAKING BIASES: RULES OF THUMB OR "HEURISTICS"

View Set

input,output, and storage review for test

View Set

FOI.6 Nucleus: Understand the storage of genetic information and how it is passed down to successive generations and the principles of basic techniques in Molecular Biology

View Set

Audit 3 Assessing risk of material Misstatement

View Set

Politics Paper 3: Global Governance: Political and Economic (2.2 Economic. 2.3, The ways and extent to which these institutions address and resolve contemporary global issues)

View Set